Все формулы по динамике физика: Ошибка: 404 Материал не найден

Содержание

Решение задач по динамике с нуля

1. Читаем ВНИМАТЕЛЬНО условие задачи.
Под действием двух ВЗАИМНО ПЕРПЕНДИКУЛЯРНЫХ сил, по модулю равных 3 Н и 4 Н, тело из состояния ПОКОЯ за 2 с переместилось на 20 м по направлению равнодействующей силы. Определить массу тела.
Обратим внимание так же на то что НЕ ДАНО, а именно ни чего не известно о ориентации в пространстве (нет указаний на вертикали или горизонтали), ни чего не известно что за силы действуют (т.е. мы НЕ ЗНАЕМ что это за силы – это какие-то безымянные силы) т.о. перед нами число абстрактная задача без привязки к какой-либо реальной ситуации!

Проанализируем условие на предмет того какие «темы работают». Так как речь о силовом воздействии то конечно – «Динамика» , так как известно время, начальная скорость и расстояние – работает так же «Кинематика»

Т.о. решение разбивается на два этапа

I) Работаем с динамикой;

II) Работаем с кинематикой;

ДИНАМИКА

I.1. Делаем рисунок и сразу показываем силы.

I.2.  Определяемся с методом, который будем использовать.  

В данном случае нам НЕ известно как направлено ускорение, и действует 2-е силы, следовательно будем использовать ГЕОМЕТРИЧЕСКИЙ МЕТОД.

I.3.  Выполним построение – ГЕОМЕТРИЧЕСКИ сложим силы (например методом параллелограмма)

I.4.  Записываем Второй закон Ньютона в векторной форме.

I.5. . Выразим численное значение равнодействующей.

Из рисунка понятно что

т.е. равнодействующую можно найти по теореме Пифагора.

С учетом II закона Ньютона можем записать

(*)   

Помним что нам нужно найти массу тела, следовательно нам не хватает ускорения тела. Найдем его из кинематики.

КИНЕМАТИКА

II.1. Известно расстояние пройденное телом (Δr = 20 м). Известна начальная скорость (V0 = 0), нужно найти ускорение – уравнение которое лучше всего «цепляет» имеющуюся информацию это уравнение перемещения

с учетом наших данных

Подставим это выражение в (*)

и выразим отсюда массу

Подставив численные значения, найдем что масса тела = 0. 5 кг.

Механика. Формулы по физике — Физика для всех

Кинематика

ОбозначениеИзмеряется вОписание
Sмпройденный путь
vм/сскорость
tсвремя
xмкоордината
aм/с2ускорение
ωс-1угловая скорость
Tспериод
νГцчастота
εс-2угловое ускорение
Rмрадиус

Скорость и ускорение:

 

Равномерное движение: ν = const

 

Равнопеременное движение: 

 

Криволинейное движение:

 

Вращательное движение:

 

Динамика и статика

ОбозначениеИзмеряется вОписание
FНсила
Pкг*м/симпульс
aм/с2ускорение
mкгмасса
vм/сскорость
pНвес тела
gм/с2ускорение свободного падения
EДжэнергия
AДжработа
NВт мощность
tсвремя
Iкг*м2момент инерции
Lкг*м2момент импульса
MН*ммомент силы
ωс-1угловая скорость

Первый закон Ньютона:
При ∑ F = 0 => v = const

Второй закон Ньютона:

 

Третий закон Ньютона:

 

Основной закон динамики для неинерциальных систем отчета.
ma=ma0+Fинерц ,где а- ускорение в неинерциальной а0— в инерциальной системе отчета.

Скорость центра масс:

 

Закон всемирного тяготения:

 

Вес тела:

 

Сила трения:

 

Закон Гука:

 

Закон Гука: σ = Eε, где Е- модуль Юнга.

 

Динамика и статика вращательного движения:

 

системаосьI
точка по окружностиось симметрииmR2
стержень через середину1/12 mR2
стержень через конец1/3 mR2
шарчерез центр шара2/5 mR2
сферачерез центр сферы2/3 mR2
кольцо или тонкостенный цилиндрось симметрииmR2
диск сплошной цилиндрось симметрии1/2 mR2

Условие равновесия тел  ∑ M = 0

Закон сохранения импульса:

 

Потенциальная и кинетическая энергия. Мощность:

 

Закон сохранения энергии:

 

Физика формулы по заданиям. Формулы по физике для егэ

Сессия приближается, и пора нам переходить от теории к практике. На выходных мы сели и подумали о том, что многим студентам было бы неплохо иметь под рукой подборку основных физических формул. Сухие формулы с объяснением: кратко, лаконично, ничего лишнего. Очень полезная штука при решении задач, знаете ли. Да и на экзамене, когда из головы может «выскочить» именно то, что накануне было жесточайше вызубрено, такая подборка сослужит отличную службу.

Больше всего задач обычно задают по трем самым популярным разделам физики. Это механика , термодинамика и молекулярная физика , электричество . Их и возьмем!

Основные формулы по физике динамика, кинематика, статика

Начнем с самого простого. Старое-доброе любимое прямолинейное и равномерное движение.

Формулы кинематики:

Конечно, не будем забывать про движение по кругу, и затем перейдем к динамике и законам Ньютона.

После динамики самое время рассмотреть условия равновесия тел и жидкостей, т.е. статику и гидростатику

Теперь приведем основные формулы по теме «Работа и энергия». Куда же нам без них!


Основные формулы молекулярной физики и термодинамики

Закончим раздел механики формулами по колебаниям и волнам и перейдем к молекулярной физике и термодинамике.

Коэффициент полезного действия, закон Гей-Люссака, уравнение Клапейрона-Менделеева — все эти милые сердцу формулы собраны ниже.

Кстати! Для всех наших читателей сейчас действует скидка 10% на .


Основные формулы по физике: электричество

Пора переходить к электричеству, хоть его и любят меньше термодинамики. Начинаем с электростатики.

И, под барабанную дробь, заканчиваем формулами для закона Ома, электромагнитной индукции и электромагнитных колебаний.

На этом все. Конечно, можно было бы привести еще целую гору формул, но это ни к чему. Когда формул становится слишком много, можно легко запутаться, а там и вовсе расплавить мозг.

Надеемся, наша шпаргалка основных формул по физике поможет решать любимые задачи быстрее и эффективнее. А если хотите уточнить что-то или не нашли нужной формулы: спросите у экспертов студенческого сервиса . Наши авторы держат в голове сотни формул и щелкают задачи, как орешки. Обращайтесь, и вскоре любая задача будет вам «по зубам».

Шпаргалка с формулами по физике для ЕГЭ

Шпаргалка с формулами по физике для ЕГЭ

И не только (может понадобиться 7, 8, 9, 10 и 11 классам). Для начала картинка, которую можно распечатать в компактном виде.

И не только (может понадобиться 7, 8, 9, 10 и 11 классам). Для начала картинка, которую можно распечатать в компактном виде.

Шпаргалка с формулами по физике для ЕГЭ и не только (может понадобиться 7, 8, 9, 10 и 11 классам).

и не только (может понадобиться 7, 8, 9, 10 и 11 классам).

А потом вордовский файл , который содержит все формулы чтобы их распечатать, которые находятся внизу статьи.

Механика

  1. Давление Р=F/S
  2. Плотность ρ=m/V
  3. Давление на глубине жидкости P=ρ∙g∙h
  4. Сила тяжести Fт=mg
  5. 5. Архимедова сила Fa=ρ ж ∙g∙Vт
  6. Уравнение движения при равноускоренном движении

X=X 0 +υ 0 ∙t+(a∙t 2)/2 S=(υ 2 —υ 0 2) /2а S=(υ +υ 0) ∙t /2

  1. Уравнение скорости при равноускоренном движении υ =υ 0 +a∙t
  2. Ускорение a=(υ υ 0)/t
  3. Скорость при движении по окружности υ =2πR/Т
  4. Центростремительное ускорение a=υ 2 /R
  5. Связь периода с частотой ν=1/T=ω/2π
  6. II закон Ньютона F=ma
  7. Закон Гука Fy=-kx
  8. Закон Всемирного тяготения F=G∙M∙m/R 2
  9. Вес тела, движущегося с ускорением а Р=m(g+a)
  10. Вес тела, движущегося с ускорением а↓ Р=m(g-a)
  11. Сила трения Fтр=µN
  12. Импульс тела p=mυ
  13. Импульс силы Ft=∆p
  14. Момент силы M=F∙ℓ
  15. Потенциальная энергия тела, поднятого над землей Eп=mgh
  16. Потенциальная энергия упруго деформированного тела Eп=kx 2 /2
  17. Кинетическая энергия тела Ek=mυ 2 /2
  18. Работа A=F∙S∙cosα
  19. Мощность N=A/t=F∙υ
  20. Коэффициент полезного действия η=Aп/Аз
  21. Период колебаний математического маятника T=2π√ℓ/g
  22. Период колебаний пружинного маятника T=2 π √m/k
  23. Уравнение гармонических колебаний Х=Хmax∙cos ωt
  24. Связь длины волны, ее скорости и периода λ= υ Т

Молекулярная физика и термодинамика

  1. Количество вещества ν=N/ Na
  2. Молярная масса М=m/ν
  3. Cр. кин. энергия молекул одноатомного газа Ek=3/2∙kT
  4. Основное уравнение МКТ P=nkT=1/3nm 0 υ 2
  5. Закон Гей — Люссака (изобарный процесс) V/T =const
  6. Закон Шарля (изохорный процесс) P/T =const
  7. Относительная влажность φ=P/P 0 ∙100%
  8. Внутр. энергия идеал. одноатомного газа U=3/2∙M/µ∙RT
  9. Работа газа A=P∙ΔV
  10. Закон Бойля — Мариотта (изотермический процесс) PV=const
  11. Количество теплоты при нагревании Q=Cm(T 2 -T 1)
  12. Количество теплоты при плавлении Q=λm
  13. Количество теплоты при парообразовании Q=Lm
  14. Количество теплоты при сгорании топлива Q=qm
  15. Уравнение состояния идеального газа PV=m/M∙RT
  16. Первый закон термодинамики ΔU=A+Q
  17. КПД тепловых двигателей η= (Q 1 — Q 2)/ Q 1
  18. КПД идеал. двигателей (цикл Карно) η= (Т 1 — Т 2)/ Т 1

Электростатика и электродинамика — формулы по физике

  1. Закон Кулона F=k∙q 1 ∙q 2 /R 2
  2. Напряженность электрического поля E=F/q
  3. Напряженность эл.
    поля точечного заряда E=k∙q/R 2
  4. Поверхностная плотность зарядов σ = q/S
  5. Напряженность эл. поля бесконечной плоскости E=2πkσ
  6. Диэлектрическая проницаемость ε=E 0 /E
  7. Потенциальная энергия взаимод. зарядов W= k∙q 1 q 2 /R
  8. Потенциал φ=W/q
  9. Потенциал точечного заряда φ=k∙q/R
  10. Напряжение U=A/q
  11. Для однородного электрического поля U=E∙d
  12. Электроемкость C=q/U
  13. Электроемкость плоского конденсатора C=S∙ε ε 0 /d
  14. Энергия заряженного конденсатора W=qU/2=q²/2С=CU²/2
  15. Сила тока I=q/t
  16. Сопротивление проводника R=ρ∙ℓ/S
  17. Закон Ома для участка цепи I=U/R
  18. Законы послед. соединения I 1 =I 2 =I, U 1 +U 2 =U, R 1 +R 2 =R
  19. Законы паралл. соед. U 1 =U 2 =U, I 1 +I 2 =I, 1/R 1 +1/R 2 =1/R
  20. Мощность электрического тока P=I∙U
  21. Закон Джоуля-Ленца Q=I 2 Rt
  22. Закон Ома для полной цепи I=ε/(R+r)
  23. Ток короткого замыкания (R=0) I=ε/r
  24. Вектор магнитной индукции B=Fmax/ℓ∙I
  25. Сила Ампера Fa=IBℓsin α
  26. Сила Лоренца Fл=Bqυsin α
  27. Магнитный поток Ф=BSсos α Ф=LI
  28. Закон электромагнитной индукции Ei=ΔФ/Δt
  29. ЭДС индукции в движ проводнике Ei=Вℓυ sinα
  30. ЭДС самоиндукции Esi=-L∙ΔI/Δt
  31. Энергия магнитного поля катушки Wм=LI 2 /2
  32. Период колебаний кол. контура T=2π ∙√LC
  33. Индуктивное сопротивление X L =ωL=2πLν
  34. Емкостное сопротивление Xc=1/ωC
  35. Действующее значение силы тока Iд=Imax/√2,
  36. Действующее значение напряжения Uд=Umax/√2
  37. Полное сопротивление Z=√(Xc-X L) 2 +R 2

Оптика

  1. Закон преломления света n 21 =n 2 /n 1 = υ 1 / υ 2
  2. Показатель преломления n 21 =sin α/sin γ
  3. Формула тонкой линзы 1/F=1/d + 1/f
  4. Оптическая сила линзы D=1/F
  5. max интерференции: Δd=kλ,
  6. min интерференции: Δd=(2k+1)λ/2
  7. Диф.решетка d∙sin φ=k λ

Квантовая физика

  1. Ф-ла Эйнштейна для фотоэффекта hν=Aвых+Ek, Ek=U з е
  2. Красная граница фотоэффекта ν к = Aвых/h
  3. Импульс фотона P=mc=h/ λ=Е/с

Физика атомного ядра

  1. Закон радиоактивного распада N=N 0 ∙2 — t / T
  2. Энергия связи атомных ядер

E CB =(Zm p +Nm n -Mя)∙c 2

СТО

  1. t=t 1 /√1-υ 2 /c 2
  2. ℓ=ℓ 0 ∙√1-υ 2 /c 2
  3. υ 2 =(υ 1 +υ)/1+ υ 1 ∙υ/c 2
  4. Е = mс 2

Единый Государственный Экзамен охватывает информацию по всему курсу физики с 7 по 11 класс. Однако если некоторые формулы по физике для ЕГЭ неплохо запоминаются сами по себе, над другими приходится поработать. Мы рассмотрим некоторые формулы, которые полезны для решения различных задач.

Кинематика

Начнем традиционно с кинематики. Частая ошибка здесь – неверное вычисление средней скорости неравномерного прямолинейного движения. В данном случае задачи пытаются решать с помощью среднего арифметического. Однако все не так просто. Среднее арифметическое – только частный случай. А для нахождения средней скорости движения существует полезная формула:

где S – весь путь, пройденный телом за определенное время t.

Молекулярно-Кинетическая Теория (МКТ)

МКТ может поставить множество коварных «ловушек» для невнимательного школьника. Чтобы избежать этого, нужно свободно владеть формулами по физике для ЕГЭ в этой области.

Начнем с закона Менделеева-Клапейрона, использующегося для идеальных газов. Он звучит так:

где p –давление газа,

V – занимаемый им объем,

n – количество газа,

R – универсальная газовая постоянная,

T – температура.

Обратите внимание на примеры задач с применением этого закона.

Все представляют себе, что такое влажность. Значения относительной влажности ежедневно сообщаются в СМИ. На экзамене же пригодится формула: здесь ф – относительная влажность воздуха,

ρ – плотность водяного пара, находящегося в воздухе,

ρ0 – плотность насыщенного пара при конкретной температуре.

Эта последняя величина – табличное значение, поэтому оно должно быть в условии задачи.

Термодинамика

Термодинамика – отрасль, достаточно близкая к МКТ, поэтому многие понятия пересекаются. Термодинамика базируется на двух своих началах. Практически каждая задача этой области требует знание и применение первого начала термодинамики, выраженного формулой

Это формулируется следующим образом:

Количество теплоты Q, которое было получено системой, расходуется на совершение работы A над внешними телами и изменение ΔU внутренней энергии данной системы.

Сила Архимеда

Напоследок поговорим о поведении погруженных в жидкость тел. Очевидно, что на каждое из них действует сила тяжести, направленная вертикально вниз. Но в жидкости все тела весят меньше. Это обусловливается частичным компенсированием силы тяжести противоположно направленной силой Архимеда. Ее значение равно Таким образом, эта сила, старающаяся вытолкнуть тело из жидкости, зависит от плотности той самой жидкости и объема погруженной в нее части тела. Сила Архимеда действует и в газах, но вследствие ничтожности плотности газов ею обыкновенно пренебрегают.

ЕГЭ проверяет знания школьника в различных областях физики. Формулы для ЕГЭ по физике способствуют успешному решению задач (можно воспользоваться ) и общему пониманию основных физических процессов.

Для того чтобы успешно подготовиться к ЦТ по физике и математике, среди прочего, необходимо выполнить три важнейших условия:

  1. Изучить все темы и выполнить все тесты и задания приведенные в учебных материалах на этом сайте. Для этого нужно всего ничего, а именно: посвящать подготовке к ЦТ по физике и математике, изучению теории и решению задач по три-четыре часа каждый день. Дело в том, что ЦТ это экзамен где мало просто знать физику или математику, нужно еще уметь быстро и без сбоев решать большое количество задач по разным темам и различной сложности. Последнему научиться можно только решив тысячи задач.
  2. Выучить все формулы и законы в физике, и формулы и методы в математике . На самом деле, выполнить это тоже очень просто, необходимых формул по физике всего около 200 штук, а по математике даже чуть меньше. В каждом из этих предметов есть около десятка стандартных методов решения задач базового уровня сложности, которые тоже вполне можно выучить, и таким образом, совершенно на автомате и без затруднений решить в нужный момент большую часть ЦТ. После этого Вам останется подумать только над самыми сложными задачами.
  3. Посетить все три этапа репетиционного тестирования по физике и математике. Каждый РТ можно посещать по два раза, чтобы прорешать оба варианта. Опять же на ЦТ, кроме умения быстро и качественно решать задачи, и знания формул и методов необходимо также уметь правильно спланировать время, распределить силы, а главное правильно заполнить бланк ответов, не перепутав ни номера ответов и задач, ни собственную фамилию. Также в ходе РТ важно привыкнуть к стилю постановки вопросов в задачах, который на ЦТ может показаться неподготовленному человеку очень непривычным.

Успешное, старательное и ответственное выполнение этих трех пунктов позволит Вам показать на ЦТ отличный результат, максимальный из того на что Вы способны.

Нашли ошибку?

Если Вы, как Вам кажется, нашли ошибку в учебных материалах, то напишите, пожалуйста, о ней на почту. Написать об ошибке можно также в социальной сети (). В письме укажите предмет (физика или математика), название либо номер темы или теста, номер задачи, или место в тексте (страницу) где по Вашему мнению есть ошибка. Также опишите в чем заключается предположительная ошибка. Ваше письмо не останется незамеченным, ошибка либо будет исправлена, либо Вам разъяснят почему это не ошибка.

Как правило, именно математику, а не физику принято считать королевой точных наук. Мы полагаем, что это утверждение спорно, ведь технический прогресс невозможен без знания физики и её развития. Из-за своей сложности она вряд ли когда-либо будет включена в список обязательных государственных экзаменов, но, так или иначе, абитуриентам технических специальностей приходится сдавать её в обязательном порядке. Труднее всего запомнить многочисленные законы и формулы по физике для ЕГЭ, именно о них мы расскажем в этой статье.

Секреты подготовки

Возможно, это связано с кажущейся сложностью предмета или популярностью профессий гуманитарного и управленческого профиля, но в 2016 году только 24 % всех абитуриентов приняли решение сдавать физику, в 2017 — лишь 16 %. Такие статистические данные невольно заставляют задуматься, не слишком ли завышены требования или просто уровень интеллекта в стране падает. Почему-то не верится, что так мало школьников 11 класса желают стать:

  • инженерами;
  • ювелирами;
  • авиаконструкторами;
  • геологами;
  • пиротехниками;
  • экологами,
  • технологами на производстве и т.д.

Знание формул и законов физики в равной степени необходимо для разработчиков интеллектуальных систем, вычислительной техники, оборудования и вооружения. При этом всё взаимосвязано. Так, например, специалисты, производящие медицинское оборудование, в своё время изучали углубленный курс атомной физики, ведь без разделения изотопов, у нас не будет ни рентгенологической аппаратуры, ни лучевой терапии. Поэтому создатели ЕГЭ постарались учесть все темы школьного курса и, кажется, не пропустили ни одной.

Те ученики, которые исправно посещали все уроки физики вплоть до последнего звонка, знают, что в период с 5 по 11 класс изучается около 450 формул. Выделить из этих четырех с половиной сотен хотя бы 50 крайне сложно, поскольку все они важны. Подобного мнения, очевидно, также придерживаются разработчики Кодификатора. Тем не менее, если вы одарены необыкновенно и не ограничены во времени, вам хватит 19 формул, ведь при желании из них можно вывести все остальные. За основу мы решили взять главные разделы:

  • механику;
  • физику молекулярную;
  • электромагнетизм и электричество;
  • оптику;
  • физику атомную.

Очевидно, что подготовка к ЕГЭ должна быть ежедневной, но если по каким-то причинам вы приступили к изучению всего материала лишь сейчас, настоящее чудо может совершить экспресс-курс, предлагаемый нашим центром. Надеемся, эти 19 формул также будут вам полезны:

Вы, наверное, заметили, что некоторые формулы по физике для сдачи ЕГЭ остались без пояснений? Мы предоставляем вам самим их изучить и открыть для себя законы, по которым абсолютно всё вершится в этом мире.

Движение проводника в магнитном поле. Электродвигатель. Динамик и микрофон. Физика, 8 класс: уроки, тесты, задания.

1. Основные понятия

Сложность: лёгкое

1
2. Действие магнитного поля на проводник с током

Сложность: лёгкое

1
3. Электродвигатель

Сложность: лёгкое

1
4. Динамик

Сложность: среднее

2
5. Микрофон

Сложность: среднее

2
6. Условные обозначения

Сложность: среднее

2
7. Четыре установки (вариант 1)

Сложность: сложное

3
8. Четыре установки (вариант 2)

Сложность: сложное

3
9. Вовочкины эксперименты

Сложность: сложное

3

Страница не найдена

владение широким спектром приёмов и способов рассуждений. Тренды в индустрии красоты Как организовать путешествие Транширование фруктов 3D-моделирование персонажа Создание сайта по образцу Модный тренд: разумное потребление Создание видеоигры Бизнес-квиз Создание коллажа будущего интерьера комнаты уверенное владение формально-оперативным алгебраическим аппаратом; умение решить комплексную задачу, включающую в себя знания из разных тем курса алгебры; умение решить планиметрическую задачу, применяя различные теоретические знания курса геометрии; умение математически грамотно и ясно записать решение, Создание авторской открытки в технике линогравюра

Урок по физике на тему «Основы динамики»

Урок – практикум.

Решение задач по теме «Основы динамики».

Цели урока: повторить с учащимися основные формулы по динамике, способы решения задач по данной теме, а также научиться решать практические задачи по данной теме.

Тип урока: урок — повторение.

План урока:

  1. Организационный момент.

2. Ознакомление учащихся с планом урока.

3. Повторительно – обобщающая беседа.

4. Работа в группах.

5. Подведение итогов урока. Постановка домашнего задания.

Ход урока:

  1. Здравствуйте, ребята, садитесь.

  2. Сегодня на уроке мы должны покинуть планету ДИНАМИКА, для того чтобы это сделать нам необходимо приобрести: 1 космическую скорость – удовлетворительно, 2 космическую скорость – хорошо, 3 космическую скорость – отлично или сверх скорость. Для этого необходимо решить соответственно 2, 3, 4 или 5 задач при этом мы продолжим знакомиться с практическим применением законов динамики, повторим и обобщим знания, имеющиеся у вас. Работать при этом будем в группах. Учет выполнения заданий будет вестись в специальной таблице на доске. Каждой группе будет выдана карточка с заданиями, как только одно задание будет выполнено, вы должны поднять флажок. Определить. правильно ли задание выполнено, можно посмотрев лист ответов. Задание, с которого вы приступите к работе, будет определено в процессе жеребьевки.

  3. Для того чтобы приступить к движению, нам нужно приобрести необходимый запас топлива, т. е. заправиться горючим – вспомнить теоретический материал.

  1. Когда возникает и как определить вес тела?

  2. Когда возникает и как определить силу тяжести?

  3. Когда возникает и как определить силу трения?

  4. Показать силы, действующие на тело, находящееся на горизонтальной поверхности?

  5. Определить, верно, ли указаны силы на рисунке, и назвать их?





4. Сейчас мы проведем жеребьевку и приступим к работе.

( Задания для групп прилагаются ).

В процессе работы, в таблице появляются флажки у тех групп, которые справились с заданием.

5. А теперь подведем итоги нашей работы, узнаем, кто сможет покинуть планету ДИНАМИКА и на следующий урок справится с контрольной работой, а кому еще придется задержаться на этой планете (по данным таблицы сообщаются результаты групповой работы).

Д
омашнее задание: повторить формулы и алгоритмы решения по динамике, задачи

№ 269, 275 по сборнику Рымкевич.

Задание для группы №1







  1. На основе имеющихся данных и рисунка,



составить систему уравнений и определить силу,


с которой тянут данное тело. m =3000 г, μ=0,005, a=1 м/с2

  1. Брусок массой 400 г, прикрепленный к динамометру, двигают равномерно по горизонтальной поверхности. Динамометр показывает при этом 1 Н. Другой раз брусок двигали по той же поверхности с ускорением. При этом динамометр показывал 2 Н. Каким было ускорение?

  2. Каков коэффициент трения при равномерном поднятии бруска вверх, по наклонной

поверхности, если длина наклонной поверхности равна 1 м, высота, на которой она

расположена 20 см, масса бруска 200 г? Силу тяги определить экспериментально при


помощи динамометра.

  1. На нити, перекинутой через неподвижный блок,

подвешены грузы массами 100 г и 200 г. Какова будет

сила натяжения нити, если поддерживать ладонью груз большей

массы, не давая системе двигаться?

  1. Брусок массой 400 г под действием груза массой 100 г проходит из состояния покоя путь10 см

за _______ секунд. Найти коэффициент трения. Определите время движения при помощи

секундомера.

_____________________________________________________________________________________

Задание для группы №2

  1. Н
    а основе имеющихся данных и рисунка составить систему уравнений и определить силу,

с которой тянут это тело.



















m =500 г, μ=0,01, a=0,5 м/с2



  1. Брусок массой 200 г, прикрепленный к динамометру, двигают равномерно по

горизонтальной поверхности. Динамометр показывает при этом 2 Н. Другой раз брусок

двигали по той же поверхности с ускорением. При этом динамометр показывал 3 Н.

Каким было ускорение?

  1. Каков коэффициент трения при равномерном поднятии бруска вверх, по наклонной

поверхности, если длина наклонной поверхности равна 1 м, высота, на которой она

расположена 30 см, масса бруска 300 г? Силу тяги определить экспериментально при

помощи динамометра.

  1. Н
    а нити, перекинутой через неподвижный блок, подвешены грузы

массами 100 г и 200 г. Какова будет сила натяжения нити,

если освободить систему и дать ей возможность

свободно двигаться?

  1. Брусок массой 400 г под действием груза массой 100 г проходит из состояния покоя путь10 см

за _______ секунд. Найти коэффициент трения. Определите время движения при помощи

секундомера.

ЕГЭ по физике. 10 советов эксперта — Учёба.ру

Онлайн-школа Учебы.ру Готовим школьников к ЕГЭ на 85+ баллов для поступления на бюджет в топовые вузы. Онлайн-занятия 2 раза в неделю — лекции и семинары в удобное время. Узнать больше

1. Выучите наизусть основные законы и формулы

В первую очередь откройте кодификатор ЕГЭ на сайте ФИПИ. В нем указан некий теоретический минимум для экзамена и кратко изложены основные законы. Для начала надо выучить наизусть все из этого минимума. Если самостоятельно можешь воспроизвести законы и формулы из кодификатора, значит, выучил.

2. Будьте последовательны

Не стоит «перепрыгивать» с электростатики на динамику, а потом на ядерные реакции и термодинамику. Изучение физики в школе начинается с классической механики. Точнее с ее первого раздела — кинематики. Вот и начинайте изучать и повторять все заново и по порядку, раздел за разделом: механика, молекулярная физика и термодинамика, электродинамика, основы специальной теории относительности и квантовая физика.

3. Решайте как можно больше задач

Чтобы хорошо понимать физику и ориентироваться в ней, нужно решать как можно больше разных задач. Установите себе планку — минимум 10 задач в день, и не отступайте от нее.

4. Внимание на единицы измерения

Решая задачу, обращайте внимание, в каких единицах измерения требуется записать ответ. Например, многие школьники привыкли писать расстояние или путь в метрах, а бывает, что ответ требуется указать в сантиметрах. Даже если решение верно, а ответ записан неправильно, результат будет нулевым.

5. Используйте задачники

Черноуцан А.И., «Физика. Задачи с ответами и решениями» — хороший задачник по всем темам. Единственный его серьезный минус — мало задач на графики, а в ЕГЭ они широко используются. Кирик Л.А., Генденштейн Л.Э., Гельфгат И.М. «1001 задача по физике с решениями» — неплохой задачник по разным уровням сложности, с подсказками.

6. Используйте рисунок

При решении задач по динамике вам поможет рисунок. Хороший чертёж на ЕГЭ по физике — залог верного ответа. С его помощью вы можете наглядно представить все силы и их направления: ничто не ускользнет от вашего внимания.

7. Уделите отдельное внимание самым сложным темам

Самыми трудными являются атомная и квантовая физика, интерференция, дифракция, фотоэффект, а также элементы ядерной физики. Это специфические темы, слабо связанные с остальными разделами предмета. Там нужно знать специальные законы и правила, что вызывает сложности.

8. Если вы не знаете, как решить задачу, все-равно запишите все законы, которые требуются для ее решения

В критериях оценки заданий № 27-№ 31 (бывшая часть С) есть интересный пункт. Если в работе записаны все необходимые законы и с ними произведены некоторые преобразования, считается, что школьник продемонстрировал действия, направленные на получение правильного ответа. А за это уже выставляется один балл из трех.

9. Решайте качественные задачи

Если на экзамене вы претендуете на максимальный балл, стоит обратить особое внимание на этот тип задач (№ 28). Существуют отдельные сборники по качественным задачам (например, «Качественные задачи по физике в средней школе», М.Е. Тульчинский). В зависимости от года издания, список рассматриваемых в этих сборниках тем может оказаться шире, чем требуется на ЕГЭ. Например, в сборниках, изданных в советское время, часто встречаются задачи на тепловое расширение, а в ЕГЭ такой темы нет. Поэтому подберите соответствующие темы по кодификатору ЕГЭ и прорешайте задачи по ним из сборника.

10. Настраивайтесь на серьезную работу

Если вы решили готовиться к ЕГЭ по физике и поступать в технический вуз — вам предстоит большой труд. Но все возможно при должном старании и регулярных занятиях.

Видео-приветствие преподавателя

Онлайн-школа Учебы.ру Готовим школьников к ЕГЭ на 85+ баллов для поступления на бюджет в топовые вузы. Онлайн-занятия 2 раза в неделю — лекции и семинары в удобное время. Узнать больше

Модуль 03: Динамика — AP Physics 1 Онлайн

Цели:


Понятия силы и результирующей силы

  • Учащийся должен уметь (а) связывать силу с движением объясните, что подразумевается под чистой или неуравновешенной силой.
Инерция и первый закон движения Ньютона
  • Учащийся должен уметь (а) сформулировать и объяснить закон Ньютона инерции (1-й закон движения) и, (b) описать инерцию и ее отношение к массе.
  • Студенты должны уметь анализировать ситуации, в которых частица остается в покое или движется с постоянной скоростью под действием нескольких сил.
Второй закон движения Ньютона
  • Учащийся должен уметь (а) сформулировать и объяснить закон Ньютона ускорения (2-й закон движения), (б) применить его к физическим ситуациях и (c) различать вес и массу.
  • Учащиеся должны понимать взаимосвязь между силой, действующей на объект, и результирующим изменением скорости объекта, чтобы они могли:
    • Вычислить, для объекта, движущегося в одном измерении, изменение скорости, которое приводит к когда постоянная сила F действует в течение заданного интервала времени.
    • Определить, для объекта, движущегося в плоскости, вектор скорости которого претерпевает заданное изменение за определенный интервал времени, средняя сила, которая действовал на объект.
  • Учащиеся должны понимать, как Второй закон Ньютона , F = ma применяется к объекту, на который действуют такие силы, как гравитация, натяжение струн или контактные силы, чтобы они могли:
    • Нарисовать колодец- маркированная диаграмма свободного тела, показывающая все реальные силы, действующие на объект.
    • Запись вниз по векторному уравнению, полученному в результате применения второго закона Ньютона к объекту, и взять компоненты этого уравнения вдоль соответствующих оси.
  • Учащиеся должны уметь анализировать ситуации в которой объект движется с заданным ускорением под действием одну или несколько сил, чтобы они могли определить величину и направление чистая сила или одна из сил, составляющих чистую силу, Например, движение вверх или вниз с постоянным ускорением.
3-й закон движения Ньютона
  • Учащиеся должны уметь (а) сформулировать и объяснить закон Ньютона закон действия-противодействия (3-й закон движения), и (б) выявить пары сил действия-противодействия.
  • Учащиеся должны понимать Третий закон Ньютона так что для данной системы они могут идентифицировать пары сил и объекты, на которые они действуют, и указать величину и направление каждого сила.
  • Учащиеся должны уметь применять третий закон Ньютона при анализе силы контакта между двумя объектами, которые ускоряются вместе по горизонтали или вертикальной линии или между двумя поверхностями, которые скользят одна по другой.
  • Студенты должны уметь решать задачи в котором применение законов Ньютона приводит к двум или трем одновременные линейные уравнения с неизвестными силами или ускорениями.
Свободные диаграммы тела и поступательное равновесие
  • Учащиеся должны уметь (а) применять законы Ньютона в анализ различных ситуаций с использованием диаграмм свободного тела, и (б) понять концепцию поступательного равновесия.
  • Студенты должны знать, что напряжение постоянна в легкой струне, проходящей через безмассовый шкив, и должен быть в состоянии использовать этот факт при анализе движения системы два объекта, соединенные строкой.
Трение
  • Учащиеся должны быть в состоянии объяснить (а) причины трение, и (б) как трение описывается с использованием коэффициентов трение.
  • Учащиеся должны понимать значение коэффициента трения , чтобы они могли:
    • Записать соотношение между нормальными силами и силами трения на поверхности.
    • Анализ ситуаций, в которых объект движется по шероховатой наклонной плоскости или горизонтальной поверхности.
    • Анализировать, при каких обстоятельствах объект начнет скользить, или рассчитать величину силы трения покоя.
  • Учащиеся должны понимать влияние сил сопротивления на движение объекта, чтобы они могли:
    • Найти конечную скорость объекта, движущегося вертикально под действием тормозящей силы, зависящей от скорости.

Устойчивое понимание 1.C:

Объекты и системы обладают свойствами инертной массы и гравитационной массы, которые экспериментально подтверждены как одинаковые и удовлетворяющие принципам сохранения.

Необходимые знания 1.C.1:

Инерционная масса — это свойство объекта или системы, определяющее, как изменяется их движение при взаимодействии с другими объектами или системами.

Цель обучения (1.C.1.1):
Учащийся может разработать эксперимент по сбору данных для определения взаимосвязи между результирующей силой, действующей на объект, его инерционной массой и его ускорением.
[см. научную практику 4.2]

Основные знания 1.C.3:

Объекты и системы обладают
свойствами инертной массы и гравитационной массы, которые экспериментально подтверждены как одинаковые и удовлетворяющие принципам сохранения.

Цель обучения (1.C.3.1):
Учащийся может разработать план сбора данных для измерения гравитационной массы и измерения инерционной массы, а также провести различие между двумя экспериментами.
[См. Научная практика 4.2]



Векторы силы

Устойчивое понимание 3.A:

Все силы имеют определенные общие характеристики, если рассматривать их наблюдателями в инерциальных системах отсчета.

Основные знания 3.A.2: 

Силы описываются векторами.

  1. Силы обнаруживаются по их влиянию на движение объекта.
  2. Силы имеют величину и направление .

Цель обучения (3.A.2.1): рисовать и вычислять

Учащийся может представлять силы на диаграммах или математически используя правильно помеченные векторы с величиной, направлением и единицами измерения во время анализа ситуации .

[См. Научную практику 1.1]

Силы заставляют вещи ускоряться

Необходимые знания 3.A.3: 

Сила , воздействующая на объект, всегда возникает из-за взаимодействия этого объекта с другим объектом.

  1. Объект не может воздействовать на самого себя.
  2. Даже если объект находится в состоянии покоя, на него могут действовать силы со стороны других объектов.
  3. Ускорение объекта, но не обязательно его скорость, всегда направлено в направлении результирующей силы, действующей на объект другими объектами.

Цель обучения (3.A.3.1):

Учащийся может анализировать сценарий и делать утверждения (разрабатывать аргументы, обосновывать утверждения) о силах, воздействующих на объект другими объектами для различных типов сил или 

компоненты сил.

[См. Научные практики 6.4 и 7.2]

Цель обучения (3.A.3.2):

Учащийся может оспорить утверждение о том, что объект может оказывать силу на себя.

[См. Научную практику 6.1]

Цель обучения (3.A.3.3):

Учащийся может описать силу как взаимодействие между двумя объектами и идентифицировать оба объекта для любой силы.

[См. Научную практику 1.4]

Третий закон движения Ньютона

Основные знания 3.A.4: 

Если один объект оказывает силу на второй объект, второй объект всегда оказывает силу равной величины на первом объекте в противоположном

направлении.

Цель обучения (3.A.4.1):

Учащийся способен строить объяснения физических ситуаций, связанных с взаимодействием тел, используя третий закон Ньютона и представление пар сил действие-противодействие.

[См. Научные практики 1.4 и 6.2]

Цель обучения (3.A.4.2):

Учащийся может использовать третий закон Ньютона, чтобы делать утверждения и прогнозировать пары действие-реакция сил при взаимодействии двух объектов.

[См. Научные практики 6. 4 и 7.2]

Цель обучения (3.A.4.3):

Учащийся может анализировать ситуации взаимодействия между несколькими объектами с помощью диаграмм свободного тела , которые включают приложение третьего закона Ньютона для определения сил.

[См. Научную практику 1.4]

Второй закон движения Ньютона

Устойчивое понимание 3.B:

Классически ускорение объекта, взаимодействующего с другими объектами, можно предсказать, используя ΣF0 = 9004 ma .

Основные знания 3.B.1:

Если интересующий объект взаимодействует с несколькими другими объектами, результирующая сила представляет собой векторную сумму отдельных сил.

Цель обучения (3.B.1.1):

Учащийся может предсказывать движение объекта под действием сил, действующих со стороны нескольких объектов, используя применение второго закона Ньютона в различных физических ситуациях с ускорением в одно измерение.

[См. Научная практика 6. 4 и 7.2]

Цель обучения (3.B.1.2):

Учащийся может разработать план сбора и анализа данных о движении (статическом, постоянном или ускоряющемся) на основе измерений силы и выполнять провести анализ, чтобы определить взаимосвязь между чистой силой и векторной суммой отдельных сил.

[См. Научные практики 4.2 и 5.1]

Цель обучения (3.B.1.3):

Учащийся способен повторно выразить диаграмму свободного тела представление в математическое представление и решить математическую задачу Представление об ускорении объекта.

[См. Научная практика 1.5 и 2.2]

Диаграммы свободного тела

Необходимые знания 3.B.2: Диаграммы свободного тела являются полезными инструментами для визуализации сил, действующих на один объект, и написания уравнений которые представляют физическую ситуацию.

  1. Объект можно нарисовать так, как если бы он был извлечен из своего окружения и идентифицировано взаимодействие с окружением.
  2. Сила, действующая на объект, может быть представлена ​​в виде стрелки, длина которой представляет величину силы, а направление указывает направление силы.
  3. Система координат с одной осью, параллельной направлению ускорения, упрощает переход от диаграммы свободного тела к алгебраическому представлению.

Цель обучения (3.B.2.1):

Учащийся может создавать и использовать диаграммы свободного тела для  анализа физических ситуаций для решения задач с движением качественно и количественно .

[См. Научная практика 1.1, 1.4 и 2.2]

Откуда берутся силы?

Прочное понимание 3.C:

На макроскопическом уровне силы можно классифицировать как дальнодействующие (действие на расстоянии) силы или контактные силы .

Необходимые знания 3.C.4:

Контактные силы возникают в результате взаимодействия одного объекта с другим объектом и возникают из-за межатомных электрических сил . Эти силы включают натяжение, трение, нормальную, упругую (физика 1) и выталкивающую силу (физика 2).

Цель обучения (3.C.4.1):

Учащийся может делать заявления о различных контактных силах между объектами на основании микроскопических причин этих сил.

[См. Научную практику 6.1]

Цель обучения (3.C.4.2):

Учащийся может объяснить контактные силы (натяжение, трение, норма, пружина) как возникающие из межатомных электрических сил и что они поэтому имеют определенные направления.

[см. научную практику 6.2]

Фундаментальные силы

Устойчивое понимание 3.G:

Определенные типы сил считаются фундаментальными.

Необходимые знания 3.G.1: 

Гравитационные силы действуют на всех масштабах и преобладают на самых больших расстояниях и массовых масштабах.

Цель обучения (3.G.1.1):

Учащийся может сформулировать ситуации,  когда гравитационная сила является доминирующей силой, а электромагнитные, слабые и сильные взаимодействия можно игнорировать.

[См. Научную практику 7.1]

Центр масс

Устойчивое понимание 4.A:

Ускорение центра масс системы связано с общей силой, действующей на систему, где ΣF = ма .

Заявление о границах: 

Физика 1 не включает вычислений центров масс; уравнение не предоставляется до физики 2. Однако ожидается, что без выполнения расчетов студенты, изучающие физику 1 , смогут определить местонахождение центра масс высокосимметричных распределений масс, таких как однородный стержень или куб с одинаковой плотностью, или две сферы равная масса.

Необходимые знания 4.A.1:

Линейное движение системы может быть описано смещением, скоростью и ускорением ее центра масс.

Цель обучения (4.A.1.1):

Учащийся может использовать представления центра масс изолированной системы из двух объектов для качественного и полуколичественного анализа движения системы.

[См. Научная практика 1.2, 1.4, 2.3 и 6.4]

Необходимые знания 4. A.2:

Ускорение равно скорости изменения скорости во времени, а скорость равна скорости изменения положения во времени.

  1. Ускорение центра масс системы прямо пропорционально суммарной силе, действующей на него со стороны всех объектов, взаимодействующих с системой, и обратно пропорционально массе системы.
  2. Сила и ускорение являются векторами, причем ускорение направлено в том же направлении, что и результирующая сила.

Цель обучения (4.A.2.1):

Ученик способен делать прогнозы о движении системы на основании того факта, что ускорение равно изменению скорости в единицу времени, а скорость равна изменению положения в единицу время.

[См. Научную практику 6.4]

Цель обучения (4.A.2.2):

Учащийся может оценить, используя данные , все ли силы на систему или все части системы были идентифицировано.

[См. Научную практику 5.3]

Цель обучения (4.A.2.3):

Учащийся способен создавать математические модели и анализировать графические соотношения для ускорения, скорости и положения центра масс системы и использовать их для расчета

свойств движения центра масс системы.

[См. Научная практика 1.4 и 2.2]

Кинематические уравнения

Необходимые знания 4.A.3:

Силы, которые системы воздействуют друг на друга, возникают из-за взаимодействия между объектами в системах. Если взаимодействующие объекты являются частями одной и той же системы, скорость центра масс этой системы не изменится.

Цель обучения (4.A.3.1):

Учащийся может применить второй закон Ньютона к системам вычислить изменение скорости центра масс при воздействии внешней силы на систему .

[см. научную практику 2.2]

Цель обучения (4.A.3.2):

Учащийся может использовать визуальные или математические представления сил между объектами в системе, чтобы предсказать, произойдет ли изменение центра- массовой скорости этой системы.

[См. Научная практика 1.4]

Физика — Динамика — Реакция на столкновение

Здесь мы рассматриваем столкновение твердых объектов и расчет реакции. Есть ряд теоретических и практических вопросов, которые нам необходимо рассмотреть, на этой странице мы начнем рассматривать уравнения, которые мы получаем в столкновениях с комбинированным вращением и линейным движением, на подстраницах более подробно рассматриваются двух- и трехмерные случаи.На этой странице обсуждается способ передачи импульса импульсом.

На этой странице мы подробнее обсуждаем практические факторы, такие как:

  • Моделируем ли мы деформацию объектов или предполагаем столкновения твердых тел.
  • Какие предположения мы делаем о коэффициенте упругости и энергии, перешедшей в тепло.
  • Какие предположения мы говорим о трении и о том, как объекты скользят.
  • Как мы работаем несколько точек соприкосновения и одновременных столкновений.

Нет правильный ответ на эти вопросы, это зависит от таких факторов, как: что мы хотим сделать, какая точность нам нужна, сколько мощности компьютера мы иметь, насколько стабильной должна быть симуляция и какой тип материалов мы имеем дело с.

Комбинированное вращение и движение с линейным движением

Трудно анализировать ситуацию, когда сталкивающиеся объекты могут свободно вращаться и двигаться прямолинейно. Основной метод, который мы будем использовать для этого, включает следующие этапы.

  1. Определите точку столкновения и направление нормали в этой точке.
  2. Рассчитать минимальный импульс для предотвращения пересечения объектов или
    изменить скорость приближения, в зависимости от коэффициента трения.
  3. Рассчитать влияние этого импульса на вращение и прямолинейное движение
    по отдельности.

Это включает в себя множество предположений и приближений, которые не всегда могут быть действительными. Нам нужно понимать, что происходит, чтобы быть уверенными, что наши симуляции или программы делают правильные предположения и что результаты будут достаточно стабильными и точными.

Этот метод основан на концепции «импульса», который представляет собой мгновенную передачу импульса между объектами. Интуитивно это неочевидно, и я думаю, что стоит потратить некоторое время на то, чтобы избавиться от наших естественных заблуждений. Столкновение может включать обмен энергией между объектами, но импульс связан с импульсом, а не с энергией, это различие означает:

  • Импульс — это векторная величина, в которой все измерения независимы, а линейные размеры не зависят от вращательных размеров.
  • Линейный и вращательный момент для замкнутой системы сохраняются независимо, то есть линейный импульс не может быть преобразован во вращательный и наоборот. Энергия может быть преобразована в другие формы.
  • Компонент импульса в каждом измерении для замкнутой системы сохраняется независимо, то есть компонент импульса x не может быть преобразован в компонент импульса y или из него. Опять же, энергия может быть преобразована в другие формы.
  • Весь импульс вызывает изменения линейного количества движения и вращательного момента, дело не в том, что часть импульса относится к линейному импульсу, а остальная часть относится к вращательному, весь импульс относится к обоим. В случае энергии мы можем приписать часть энергии прямолинейному движению, а другую часть вращательному движению, и энергия может передаваться между ними.

Этот последний пункт трудно понять людям, мне потребовалось много времени, чтобы понять это, и я обнаружил, что у многих других людей есть проблемы с этим. Это кажется чем-то напрасным, один и тот же импульс выполняет две работы, но это не так. Импульс просто обменивается импульсом между двумя объектами, и один и тот же импульс может передавать два типа импульса.Это не то же самое, что энергия, и работает совершенно по-другому.

Если вы хотите увидеть практический пример этого, см. эту страницу (присланную мне Кевином Пегрумом), которая демонстрирует это и выделяет некоторые противоречащие интуиции проблемы.

Результаты

Вот уравнения для расчета компонентов импульса (J), а затем уравнения для их использования для расчета новых скоростей. Это всего лишь обзор, определения и выводы даны на отдельных страницах для одного, двух и трех случаев:

Результаты для одномерного случая (без вращения) следующие: (см. эта страница для кода и происхождения)

Результаты для двумерного случая следующие: (см. страница для кода и вывода)

  Общий случай (между Совершенно эластичный и Совершенно неэластичный)
импульс =

Jx = (e+1)/k * (Vaix — Vbix)( 1/ma — rax 2 /Ia + 1/mb — rbx 2 /Ib) — (e+1)/k * (Vaiy — Vbiy) (rax ray / Ia + rbx rby / Ib)
Jy = — (e+1)/k * (Vaix — Vbix) (rax луч / Ia + rbx rby / Ib) + (e+1)/k * (Ваий — Вбий) ( 1/ма — луч 2 /Я + 1/мб — рбы 2 /Иб)

где:

k=1/ma 2 + 2/ma mb +1/mb 2 — rax 2 /maIa — rbx 2 /maIb — луч 2 /maIa — луч 2 /mbIa — rax 2 /mbIa — rbx 2 /mbIb — rby 2 /maIb — rby 2 /mbIb + ray 2 rbx 2 /IaIb + rax 2 rby 2 /IaIb — 2 Rax Ray RBX RBY/IAIB

Конечная скорость объекта a= af = Вайкс — Jx/Ма
Вай — Ян / Ма
Конечная скорость объекта b= bf = Вбикс — Джкс/Мб
Vbiy — Jy/Mb
Конечная угловая скорость объекта a=w af =

waix — (Jx ray — Jy rax) /I
waiy — (Jx ray — Jy rax) /I

Конечная угловая скорость объекта b=w bf = wbix — (Jx rby — Jy rbx) /I
wbiy — (Jx rby — Jy rbx) /I

Результаты для трехмерного случая следующие: (см. страница для кода и вывода)

  Общий случай (между Совершенно эластичный и Совершенно неэластичный)
импульс Дж =
-(1+е) (v a -v b )•n + (r a ×n)•ω a — (r b ×n)•ω b
1/m a +1/m b +(ra×n)•([I a ] -1 (r a ×n))+(r b × n)•([I b ] -1 (r b ×n))
Конечная скорость объекта a= af = Вай — Дж/млн
Конечная скорость объекта b= bf = Вби — Дж/Мб
Конечная угловая скорость объекта a=w af = вай — [Ia] -1 (J x ra)
Конечная угловая скорость объекта b=w bf = wbi — [Ib] -1 (J x rb)

Реакция на столкновение для незакрепленных твердых тел

До сих пор мы разделяли компоненты вращения и линейные компоненты, было бы хорошо, если бы мы могли объединить их в один набор уравнений, я могу придумать два подхода: здесь мы объединяем их с помощью векторов и матриц. Я также хотел бы попробовать подход с использованием геометрической алгебры и конформного пространства, но я пока не добился никакого прогресса.

Как уже объяснялось здесь, при отсутствии ограничений твердые тела плавают в свободном пространстве, возможно, под влиянием силы, то каждый объект может быть представлен вектором состояния, как показано ниже:

Вектор состояния динамики для объекта:

θx
θу
θз
пикселей
Ру
Пз
люкс
лай
лз
МВХ
мвы
мвз

Эти значения могут постепенно изменяться со временем таким образом, который можно описать простыми линейными уравнениями. Это будет продолжаться до тех пор, пока два или более объектов не столкнутся. это приводит к ступенчатому изменению этих переменных, разрыву или сингулярности, когда импульс передается между двумя объектами в бесконечно малом мало времени. Когда происходит такое столкновение, нам нужно вычислить новое состояние вектора для каждого объекта. Для этого нам необходимо собрать следующую информацию который описывает столкновение:

Параметры столкновения:

и ссылка на объект
б ​​ ссылка на объект b
сва вектор состояния для объекта a — 12-мерный вектор, как указано выше
свб вектор состояния для объекта b — 12-мерный вектор, как указано выше
Матрица Ньютона-Эйлера для объекта а
айакс иакси иакс 0 -газ сено
иайкс иауу иайз газ 0 -хах
азкс иазы иаз -сено гах 0
0 газ -сено мА 0 0
-газ 0 гах 0 мА 0
сено -хах 0 0 0 мА

Эта матрица объясняется здесь и объединяет следующую информацию:

  • масса объекта а
  • Тензор инерции для объекта а (в глобальной системе координат — так необходимо вычислить во время столкновения)
  • смещение точки столкновения от центра масс объекта а — также необходимо рассчитывать в момент столкновения.
Матрица Ньютона-Эйлера для объекта b
ibxx икси ибз 0 -хбз по
ибис ибый ибыз хбз 0 -хбх
ибзкс ибзы ибзз -hby хбх 0
0 хбз -hby мб 0 0
-хбз 0 хбх 0 Мб 0
по -хбх 0 0 0 Мб

См. выше для объекта b

нет Вектор нормали (размерность 3), это нормаль к точке столкновения и показывает направление, в котором передается импульс.
cfa, cfb Дополнительный коэффициент трения для каждого объекта

Предположения по физике

Ниже приводится аналитический подход к столкновениям, он предполагает, что столкновение происходит при точечном контакте между твердыми телами. Это означает, что объекты не ломаются и не деформируются при столкновении. Если бы эти предположения вызвали слишком много ошибки, например, моделирование деформации автомобиля при аварии, или если аналитический подход приводит к слишком сложным уравнениям, тогда попробуйте численный подход к столкновениям.

Уравнения динамики для одного объекта были рассмотрены в предыдущем страница. Здесь мы распространяем этот анализ на два объекта, чтобы мы могли вычислить результат столкновения объектов. Как объяснялось в предыдущем страницы, нам нужно перевести все значения (инерция, сила, движение и т.д.) в мир или общие координаты.

Линейный импульс, угловой момент и энергия передается между сталкивающимися формы в точке столкновения с помощью импульса.Предполагая, что мы знаем скорости перед ударом и точка удара, затем один или несколько из следующего может быть использовано, чтобы помочь нам вычислить скорости после столкновение:

  • Сохранение импульса — Суммарный импульс всей системы будет законсервированный. Импульс является векторной величиной, поэтому в трехмерном случае он сохраняется. по любой оси x, y и z, которую мы выберем.
  • Сохранение энергии — будет сохраняться полная энергия всей системы, но энергия может быть преобразована между кинетической энергией и другими видами энергии.Таким образом, мы должны сделать предположения об этом. При абсолютно упругом столкновении то кинетическая энергия сохраняется, при неупругом столкновении часть энергия, связанная с приближающимися объектами, может быть преобразована в тепло.
  • Расчет импульса — можно рассчитать импульс, что позволит нам вычислить изменение скорости обоих тел.

Также есть разные исходы в зависимости от свойств объектов столкновение:

  • Если объекты абсолютно эластичны, они будут поглощать кинетическую энергию столкновения как потенциальную энергию (как пружина), а затем мгновенно преобразовать обратно к кинетической энергии, так что объекты отскакивают друг от друга с разделяющая скорость равна и противоположна приближающейся скорости.
  • Если объекты абсолютно неэластичны, они слипнутся, и конечные скорости двух объектов будут равны. В этом случае общая кинетическая энергия будет уменьшена, а часть энергии будет преобразована в тепло или другие виды энергии.
  • Объекты могут находиться где-то посередине между этими двумя крайностями. Так что мы могли ввести коэффициент упругости для каждого сталкивающегося объекта, который дал бы конечные скорости где-то между этими двумя крайностями. Еще одно осложнение заключается в том, что этот фактор может быть разным в разных измерениях, например объекты могут сталкиваться и скользить друг относительно друга, поэтому может потребоваться трение. принять во внимание.

Другие концепции:

Импульс есть интеграл силы по время измеряется в ньютон-секундах. Например, сила в один ньютон приложенная в течение одной секунды, изменит импульс, сила в два Ньютона применение в течение половины секунды будет иметь тот же эффект.Для жесткого тела столкновений, мы доводим это до предела и применяем бесконечную силу к бесконечно малое время. Этот импульс равен изменению импульса сталкивающихся объектов. Поскольку мы говорим здесь о силах, Ньютон действует третий закон, и импульс на сталкивающиеся объекты будет равен и наоборот.

Вращение

Как только мы начнем работать в двух и трех измерениях, появится возможность вращения включить. Это делает общее решение столкновения очень сложно выработать (слишком сложно для меня пока — но я работаю на нем, вы можете помочь?). Вращение имеет те же законы, что и линейные столкновения. существуют эквиваленты законам Ньютона для вращений и полного углового импульс системы сохраняется так же, как линейный импульс законсервированный. Однако следующие проблемы делают расчет столкновения ответ затруднен:

  • Много других переменных.В 3-х измерениях 12 степеней свободы, т.е. 12 переменных для расчета, 3 линейных для каждого объекта и 3 угловых для каждого объекта.
  • При работе с линейными величинами в ньютоновской системе существует принцип «относительности», то есть скорости измеряются относительно чего-то иначе, или говоря иначе, система отсчета может быть установлена ​​относительной к любому из объектов в системе и все правила будут работать корректно. На самом деле нет абсолютных скоростей, нет системы отсчета лучше чем любой другой. Итак, мы можем путешествовать на одном из подвижных объектов и рассчитать реакцию сталкивающегося объекта относительно него. в В случае системы с вращением существует абсолютная система отсчета. для вращения это система отсчета, в которой объекты без силы, действующие на них, движутся прямолинейно.В противном случае, если у нас есть вращающаяся система отсчета, скажем, с точки зрения человека стоя на вращающейся планете, вы увидите появление сталкивающегося объекта. путешествовать по спирали. Таким образом, законы Ньютона не работают во вращающемся точка зрения.
  • Существуют и другие усложнения в системе смешанных линейно-вращательных движения, например, результирующие скорости зависят от точки воздействия, т.е.е. объекты столкнулись лоб в лоб, или это скользящее столкновение.

Трение

При сложном столкновении может иметь место трение. Однако трение (в отличие от эластичности) участвует только в том случае, если есть сила, такая как гравитация, толкая сталкивающиеся объекты вместе.

 

Коллизии в 3-х измерениях довольно сложная проблема, начну с 1-го измерения сначала попробуйте построить до 2-х, а потом и до 3-х измерений.


Реализация этого в программе

Одна проблема, которую я хотел бы иметь в виду на этих страницах, это то, как мы используем эта теория в программе? Предлагается включить информацию об этих физические свойства в графе сцены.

Информацию о том, как использовать это в программе см. в динамике руководство.


Определение точки удара

Нам нужно не только обнаружить столкновение, но и найти точку удара относительно центров масс, чтобы мы могли пройти эта информация поступает в систему реагирования на столкновение.


Следующий шаг и дополнительная литература

На этой странице предполагается, что столкновение каким-то образом было обнаружено. обсуждается здесь.

Чтобы увидеть, как алгоритм реализации реакции на столкновение твердого объекта глянь сюда.

Чтобы увидеть предложение по структуре для хранения этой информации о реакции на столкновение глянь сюда.

Вот вопрос не по теме, вроде бы есть 1 способ, которым тело может вращаться в 2 измерения и 3 способа вращения тела в 3 измерениях.Как Сколькими способами тело может вращаться в 4-х измерениях?

Каждая таблица на листе уравнений AP Physics 1 с объяснением

Отличительной особенностью экзамена AP Physics 1 является то, что сдающих экзамен имеют доступ к таблице уравнений и формул, на которые можно ссылаться во время экзамена (которую часто называют «таблицей уравнений AP Physics 1»).

Но справочные таблицы AP Physics 1 содержат много информации! Если вы еще не ознакомились с таблицей формул перед сдачей экзамена, вы можете потратить драгоценное время, пытаясь ориентироваться в различных уравнениях и помнить, когда и как их использовать.

Чтобы помочь вам, , мы разработали таблицу уравнений PrepScholar Physics 1. Этот лист содержит все уравнения, которые вы увидите на фактическом листе уравнений AP Physics 1 , а также дополнительных пояснений, которые помогут вам использовать его в качестве учебного пособия.

В оставшейся части этой статьи, , мы дадим вам подробное объяснение каждой таблицы информации, представленной на листе уравнений AP Physics 1, и объясним, как ее можно использовать на экзамене .Мы также дадим вам три совета по использованию листа формул на экзамене и три совета по использованию листа формул для подготовки к экзамену.

 

2021 Изменения в тестах AP в связи с COVID-19

Из-за продолжающейся пандемии коронавируса COVID-19 тесты AP теперь будут проводиться в течение трех разных сессий в период с мая по июнь. Даты ваших тестов, а также то, будут ли ваши тесты онлайн или бумажными, будут зависеть от вашей школы. Чтобы узнать больше о том, как все это будет работать, и получить самую свежую информацию о датах тестирования, онлайн-обзоре AP и о том, что эти изменения означают для вас, обязательно ознакомьтесь с нашей статьей часто задаваемых вопросов AP COVID-19 2021 года.

 

Что вы увидите на экзамене по физике AP 1? Вопросы по электричеству!

 

Экзамен AP Physics 1

Экзамен AP Physics 1 — это основанный на алгебре экзамен, который оценивает понимание экзаменуемым кинематики, динамики, кругового движения и гравитации, энергии, импульса, простого гармонического движения, крутящего момента и вращательного движения, электрического заряда. электрическая сила, цепи постоянного тока, механические волны и звук. По сути, экзамен AP Physics 1 проверяет ваше понимание основных концепций классической механики!

Этот экзамен AP длится три часа и включает 50 вопросов с несколькими вариантами ответов и пять вопросов с бесплатным ответом , при этом каждый раздел оценивается в 50% от общей оценки экзамена. Раздел с множественным выбором длится 90 минут, а 50 вопросов этой части теста разделены на два подраздела. Вот как они разбиваются:

Секция

Количество вопросов

45 вопросов с несколькими вариантами ответов

5 вопросов с несколькими вариантами ответов

Пять вопросов с бесплатным ответом длятся 90 минут, и тема каждого отдельного вопроса с бесплатным ответом следующая:

Номер вопроса

Вопрос Тема/Формат

Вопрос 1

Экспериментальный дизайн

Вопрос 2

Качественный/количественный перевод

Вопрос 3

Аргумент абзаца/краткий ответ

Вопрос 4

Краткий ответ

Вопрос 5

Краткий ответ

 

Лист уравнений AP Physics 1 будет включен в ваш экзаменационный буклет в день экзамена, и вы сможете использовать его в качестве справочного материала в течение экзаменационного периода .

Так как нам нужно охватить так много, мы составили специальную версию таблицы уравнений для PrepScholar. Он содержит всю информацию, которую вы увидите на исходном листе уравнений, а также пояснения к каждому уравнению. Мы будем использовать эту таблицу уравнений в оставшейся части документа, поэтому обязательно загрузите ее сейчас.

Далее мы более подробно рассмотрим каждую таблицу информации, представленную на листе формул AP Physics 1.

Это официальный формуляр AP Physics 1, который вы получите в день экзамена.

 

Таблица формул AP Physics 1 с объяснением

Таблица формул AP Physics 1 является ключевым ресурсом для ответов на вопросы этого экзамена по алгебре. Копия листа с уравнениями будет предоставлена ​​в вашем экзаменационном буклете во время экзамена (вы не можете принести свои копии в экзаменационную аудиторию), и включает общие уравнения, которые рассматриваются в курсе AP Physics 1.

Если вы еще этого не сделали, обязательно загрузите формуляр PrepScholar, который можно использовать в качестве учебного пособия. В день экзамена вам выдадут чистую копию официального учебного листа, но мы рекомендуем вам распечатать копию, которую вы сможете разметить и использовать во время учебы!

Лист уравнений AP Physics 1 состоит из таблиц на основе следующих типов информации:

  • Константы и коэффициенты пересчета (стр. 1)
  • Символы единиц измерения (стр. 1)
  • Префиксы (стр. 1)
  • Значения тригонометрических функций для обычных углов (стр. 1)
  • Уравнения, обычно используемые в физике для механики, электричества, волн, геометрии и тригонометрии (стр. 2)

Лист уравнений предназначен для того, чтобы помочь вам быстро вспомнить константы, коэффициенты преобразования, символы, префиксы, значения и уравнения, которые могут вам понадобиться для решения задач во время экзамена. Важно помнить, что каждое уравнение, которое вы используете из листа уравнений, должно сопровождаться пояснениями и логическим развитием ваших ответов на экзамене. Это означает, что вам придется хорошо понимать формулы и то, как их использовать, если вы хотите успешно пройти тест AP Physics 1!

 

Как использовать формулы на листе уравнений AP Physics 1

Чтобы помочь вам ознакомиться с тем, как использовать лист уравнений AP Physics 1, мы разберем, как использовать следующие области листа уравнений по отдельности. В частности, мы рассмотрим следующие темы:

  • Константы и коэффициенты пересчета
  • Префиксы и символы единиц измерения
  • Значения тригонометрических функций
  • Уравнения механики, электричества, волн, геометрии и тригонометрии.

Давайте взглянем на основные разделы таблицы уравнений физики 1.

 

 

Константы и коэффициенты пересчета

Константы и коэффициенты преобразования отображаются в верхней части первой страницы листа уравнений, который вы будете использовать на экзамене AP Physics 1. Это фиксированные значения, которые вам необходимо знать и использовать в формулах и уравнениях на экзамене.

Константы и коэффициенты пересчета, приведенные в информационном листе AP Physics 1, включают массу протона, массу нейтрона, массу электрона, скорость света, величину заряда электрона, постоянную закона Кулона, универсальную гравитационную постоянную и ускорение под действием силы тяжести на поверхности Земли.

Итак, как вы будете использовать эти переводные коэффициенты в день экзамена? Константы и коэффициенты преобразования можно использовать на экзамене для преобразования одной единицы измерения в другую с помощью умножения или деления. Это изменит единицы измерения без изменения значения этого измерения. Коэффициенты преобразования, представленные на листе уравнений, можно использовать для преобразования длины, массы, времени, энергии, температуры, частоты, силы, мощности, заряда и сопротивления.

 

 

Символы единиц измерения, префиксы и значения тригонометрических функций

Таблицы символов единиц и префиксов можно комбинировать для выражения значений на экзамене AP Physics 1 . Таблица префиксов обеспечивает научное обозначение или фактор данного префикса, префикса и соответствующего символа.

Звучит запутанно, но вот что мы имеем в виду. Например, в таблице указан префикс «тера» вместе с правильным коэффициентом 10 12 и правильным символом «Т». Точно так же таблица символов единиц содержит название единицы и ее правильный символ , например, «метр» и «м» или «кельвин» и «К».

Префиксы, включенные в информационный лист, используются при работе с очень большими или маленькими единицами в вопросах экзамена .Префиксы указывают конкретные степени числа десять и обычно используются для выражения измерений в сочетании с базовым словом из таблицы символов единиц (например, киловатты, мегаджоули и т. д.). Эта часть таблицы поможет вам лучше понять вопросы экзамена, а также поможет перепроверить, чтобы убедиться, что вы используете правильные единицы измерения в своих ответах на вопросы со свободным ответом.

Наконец, значения тригонометрических функций будут иметь решающее значение при использовании геометрических и тригонометрических уравнений для расчета значений углов прямоугольного треугольника .Эта таблица выражает значение наиболее распространенных углов (sin, cos, tan) в различных градусах вплоть до угла 90 градусов. Вам нужно будет понять их, чтобы делать такие вещи, как анализ векторов!

 

Одной из наиболее важных частей вашего заявления в колледж является то, какие предметы вы выберете в старшей школе (в сочетании с тем, насколько хорошо вы успеваете по этим предметам). Наша команда экспертов по приему в PrepScholar собрала свои знания в этом едином руководстве по планированию расписания занятий в старшей школе. Мы посоветуем вам, как сбалансировать свое расписание между обычными курсами и курсами с отличием/AP/IB, как выбрать дополнительные занятия и какие занятия вы не можете позволить себе не посещать.

 

Уравнения

На второй странице листа формул AP Physics 1, предоставленного на экзамене, содержится список общих уравнений, которые вам могут понадобиться на экзамене. Уравнения разделены на четыре раздела в зависимости от типа: механика, электричество, волны, геометрия и тригонометрия.

Ниже мы объясним, какие типы задач помогут вам решить уравнения, включенные в каждый раздел листа уравнений.

 

 

Таблица механики

Уравнения в таблице Mechanics можно использовать для расчета, описания, анализа, выражения, объяснения, а также для утверждения и прогнозирования следующего на экзамене AP Physics 1:

  • Ускорение, включая радиальное ускорение, тангенциальное ускорение и ускорение объекта, взаимодействующего с другими объектами

  • Движение, включая линейное движение, угловое движение и движение отдельных объектов и систем из двух объектов

  • Силы, включая контактные силы между объектами, такие как силы натяжения, трения, нормальные, плавучие и пружинные,

  • Гравитационная сила, включая гравитационную силу, с которой два объекта действуют друг на друга

  • Гравитационная сила в различных контекстах

  • Изменение кинетической энергии, расчет полной энергии системы, прогнозирование изменений полной энергии системы, расчет внутренней потенциальной энергии, расчет мощности

  • Импульс, угловой момент, величина углового момента, изменение углового момента

  • Крутящий момент

 

 

Электрический стол

Уравнения в гораздо более короткой таблице электричества на листе уравнений можно использовать для расчета и описания следующего на экзамене AP:

  • Величина электрического поля
  • Сохранение электрического заряда
  • Удельное сопротивление вещества
  • Сохранение электрического заряда в электрических цепях

 

 

Волновой стол

В листе формул AP Physics 1 есть одно уравнение, относящееся к волнам; это уравнение можно использовать для расчета длины волны периодической волны.

 

 

Таблица геометрических и тригонометрических уравнений

Наконец, последний раздел листа уравнений содержит геометрические и тригонометрические уравнения, которые можно использовать для решения следующих задач:

  • Площадь прямоугольника
  • Площадь треугольника
  • Площадь и длина окружности
  • Объем прямоугольного твердого тела
  • Объем и площадь поверхности цилиндра
  • Объем и площадь поверхности шара
  • Величина углов прямоугольного треугольника

Поскольку в справочных таблицах AP Physics 1 содержится так много формул и уравнений, стоит потратить некоторое время на ознакомление с ними перед сдачей экзамена. О том, как лучше всего ознакомиться с содержанием таблицы уравнений, мы поговорим ниже.

 

 

3 совета по использованию таблицы формул AP Physics 1 в качестве учебного пособия

Поскольку справочные таблицы AP Physics 1 будут доступны вам на самом экзамене, вы можете заранее воспользоваться этим ресурсом, используя его для подготовки к экзамену. Ознакомьтесь с нашими тремя советами по обучению с формулой AP Physics 1 ниже!

 

Учебный совет 1: сделайте карточки с уравнениями

Практически гарантировано, что уравнения, приведенные в листе формул AP Physics 1, появятся на экзамене. Хотя эти уравнения будут у вас под рукой во время сдачи экзамена, вы не хотите тратить драгоценное экзаменационное время на их расшифровку.

На листе уравнений есть ключ к символу, который поможет вам расшифровать, что означает каждый символ в данном уравнении, но у вас будет больше времени, чтобы точно ответить на вопросы экзамена, если вам не нужно использовать эту часть экзаменационного листа. на протяжении всего экзамена.

Вместо для подготовки к экзамену используйте лист уравнений AP Physics 1 для создания карточек, которые помогут вам запомнить уравнения. Чтобы использовать лист уравнений в своих интересах во время учебы, на каждой из ваших карточек должно быть уравнение из листа уравнений с одной стороны и ключ, который разбивает каждую переменную в уравнении на противоположной стороне. Если вы уже знаете, что означает « v » или « K » или «U» в каждом уравнении на экзамене, вам не придется тратить время, используя таблицу уравнений, чтобы разбить каждую переменную. в уравнении, которое вам нужно использовать для решения проблемы.

 

Нужна помощь в подготовке к экзамену AP?

Наши индивидуальные услуги онлайн-репетиторов AP помогут вам подготовиться к экзаменам AP. Найди лучшего репетитора, получившего высокий балл на экзамене, к которому ты готовишься!

 

Совет по изучению 2: пройдите пробный тест

Вероятно, лучший способ понять, как лучше всего использовать лист формул AP Physics 1 на настоящем экзамене, — это пройти пробный тест или, по крайней мере, ответить на серию практических вопросов, используя лист в качестве ресурса.

Несмотря на то, что практических экзаменов AP Physics 1 не так много, существует неофициальный экзамен, доступный в CrackAP. Вы также можете просмотреть FRQ прошлых экзаменов, которые можно найти на веб-сайте College Board.

Во время практики, , подумайте о том, чтобы отметить, когда вам больше всего нужно обратиться к листу уравнений, а затем потратить дополнительное время на изучение заметок или карточек , которые у вас есть, которые относятся к этим областям. Это поможет вам определить свои слабые стороны и укрепить их перед сдачей экзамена AP.

 

Совет по изучению 3: запомните макет

Если вы доберетесь до экзамена и еще не знакомы с макетом листа уравнений и с тем, какие уравнения включены, а какие нет, будет довольно сложно использовать лист в своих интересах во время экзамена.

Потратив некоторое время на то, чтобы запомнить, какая информация находится где на листе уравнений, и общее представление о том, какие уравнения и информация включены в лист, помогут вам точно знать, когда вы можете обратиться к листу уравнений для получения информации или напоминаний, как вы сдаете экзамен . Запоминание информационного листа поможет вам быть более эффективным и организованным при сдаче экзамена AP Physics 1.

 

Как мы уже говорили, лучше всего использовать лист уравнений AP Physics 1, чтобы помочь вам выучить важные уравнения, которые вам нужно знать в день экзамена. Но на случай, если вы этого не сделали, вот наши лучшие советы по использованию листа на реальном экзамене.

 

3 совета по использованию таблицы уравнений AP Physics 1 в день экзамена

Поскольку вам разрешено использовать PDF-файл CollegeBoard листа формул AP Physics в день экзамена, вы хотите убедиться, что знаете как использовать этот лист в своих интересах, пока вы фактически сдаете экзамен.Прочтите наши три совета по использованию листа формул AP Physics в день экзамена!

 

Совет 1: сэкономьте свое время

Так как экзамен AP Physics 1 рассчитан на время, вы действительно не хотите тратить больше времени, чем это абсолютно необходимо, пытаясь запомнить значения, формулы и уравнения во время экзамена. Если вы застряли и просто не можете вспомнить значение или часть уравнения, которое имеет решающее значение для ответа на вопрос, быстрое перелистывание листа с уравнениями может помочь вашей памяти.

 

Совет 2. Делайте быстрые преобразования

Константы и коэффициенты преобразования, которые обычно используются в физических задачах, немного сложны. Обычно они включают в себя несколько десятичных знаков, показатели степени и другие символы, которые может быть трудно запомнить перед экзаменом. Лист уравнений будет полезен для быстрого выполнения преобразований и запоминания правильных выражений для общих констант при решении задач на тесте.

 

Совет 3: проверьте свою работу

При ответах на вопросы экзамена AP Physics 1 крайне важно уделять внимание деталям.Но это может быть сложно в тесте на время, и, вероятно, легче случайно забыть включить символ, показатель степени или обозначение, чем вы думаете. Если вы потратите несколько минут на проверку своей работы с помощью листа уравнений во время экзамена, это поможет вам внести исправления и убедиться, что вы правильно написали формулы и уравнения, особенно в вопросах с бесплатными ответами.

 

 

Что дальше?

В этой статье рассматривается лист уравнений по физике 1, но знаете ли вы, что вы можете пройти два других курса физики AP, пока вы учитесь в старшей школе ? Узнайте больше об AP Physics 1, 2 и C и о различиях между ними.

Если вам нужны ресурсы для курсов IB Physics, у нас они тоже есть . Вот экспертное руководство по программе IB Physics. Мы также составили список лучших учебных заметок по физике SL и HL.

Вам может быть интересно, насколько сложна AP Physics 1 на самом деле. Чтобы получить ответ, ознакомьтесь с этой статьей, которая поможет вам понять, какие классы AP для вас самые сложные.

 

Хотите улучшить свой результат SAT на 160 баллов или свой результат ACT на 4 балла? Для каждого теста мы составили руководство по 5 основным стратегиям, которые вы должны использовать, чтобы попытаться улучшить свой результат. Скачать бесплатно прямо сейчас:

 

Физические уравнения MCAT, которые вы должны знать в 2022 году

Сколько физики на MCAT?

Вам может быть интересно, сколько физики вы увидите на MCAT? Ваши знания по физике пригодятся в первом разделе MCAT: Химические и физические основы биологических систем. По данным AAMC, вы можете ожидать, что примерно 25% вопросов в этом разделе будут относиться к вводной физике.

Сколько вводной физики на MCAT?

Что мы подразумеваем под введением в физику? Вы не будете использовать слишком сложные физические уравнения в этом разделе MCAT, а скорее вам нужно будет уметь применять физические концепции из вашего двухсеместрового вводного курса университетской физики, чтобы продемонстрировать широкое понимание динамики в живых системах. . Вы можете ожидать увидеть вопросы, связанные с физикой, основанные на отрывках, а также несколько отдельных вопросов по дискретной физике.Когда начинать подготовку к MCAT и как готовиться к MCAT, будет частично зависеть от того, сколько знаний вы сохранили на вводных курсах физики.

AAMC определил ваше понимание того, как сложные живые организмы транспортируют материалы, воспринимают окружающую среду, обрабатывают сигналы и реагируют на изменения — с точки зрения физических принципов — в качестве основополагающей концепции MCAT. Приблизительно 40 % раздела химии и физики будет сосредоточено на этой основополагающей концепции и будет включать следующие категории содержания, связанные с физикой:

4A – Поступательное движение, силы, работа, энергия и равновесие в живых системах

4B – Важность жидкостей для кровообращения, движения газов и газообмена

4C – Электрохимия и электрические цепи и их элементы

4D – Как свет и звук взаимодействуют с материей

4E – Атомы, ядерный распад, электронное строение и атомно-химическое поведение

Подробно изучите категории контента на MCAT с помощью руководства AAMC «Что входит в экзамен MCAT?»

Основные физические уравнения для MCAT

Существует множество физических уравнений, но какие из них вам действительно нужно знать для MCAT? Продолжайте читать, чтобы ознакомиться с каждым уравнением физики, которое AAMC рекомендует вам знать, с разбивкой по категориям контента:

4A — Поступательное движение, силы, работа, энергия и равновесие в живых системах

Эта категория контента посвящена движение и его причины, а также различные формы энергии и их взаимопревращения.

1. Второй закон Ньютона: F = ma

  • Это уравнение является вторым законом Ньютона, который утверждает, что результирующая сила (F), действующая на объект, пропорциональна массе объекта (m) и ускорению (a).

2. Работа постоянной силы: W = Fd cosθ

  • Это уравнение описывает принцип работы энергии, или работу (W), совершаемую постоянной силой (F) над объектом, который движется в определенном направлении . В этом уравнении d — это расстояние, на которое перемещается объект, пока на него действует сила, а косинус тета (cosθ) — это угол между силой и смещенным объектом.

3. Теорема о кинетической энергии работы: Wnet = ΔKE

  • Эта теорема утверждает, что чистая работа (Wnet) системы равна изменению кинетической энергии (ΔKE) движущегося объекта, частицы или системы объекты движутся вместе.

4. Кинетическая энергия: KE = ½ mv 2

  • Кинетическая энергия (KE) – это форма энергии, связанная с движением объекта. Эта энергия связана с определенной массой (m), движущейся с определенной скоростью (v).Кинетическая энергия пропорциональна квадрату скорости (v 2 ).

5. Потенциальная энергия: PE = mgh

  • Это уравнение описывает гравитационную потенциальную энергию (PE), которая зависит от положения объекта. Чтобы использовать это уравнение, вам понадобится масса объекта (м), ускорение свободного падения (g), которое составляет 9,8 м/с 2 на поверхности Земли, и высота объекта в метрах (h) .

6. Потенциальная энергия: PE=½kx 2

  • Сила упругости – это сила, возникающая в результате растяжения или сжатия объекта, например пружины.В этом уравнении потенциальной энергии (PE) k — это постоянная пружины, а x — расстояние, на которое растягивается пружина. Постоянная пружины относится к жесткости пружины.

4B – Важность жидкостей для циркуляции крови, движения газов и газообмена

Эта категория контента посвящена поведению жидкостей, так как оно имеет отношение к функционированию легочной и сердечно-сосудистой систем.

1. Закон Паскаля о гидростатическом давлении: P = ρgh

  • Этот закон применим к статическим жидкостям и связывает давление с глубиной.Давление в жидкости на данной глубине называется гидростатическим давлением, и это давление увеличивается по мере увеличения глубины под поверхностью. В этом уравнении P — гидростатическое давление, ρ — плотность жидкости, g — ускорение свободного падения (9,8 м/с 2 ), а h — глубина/высота жидкости в метрах.

2. Уравнение непрерывности: A∙v = константа

  • Непрерывность потока является фундаментальным принципом жидкостей. Поскольку масса в жидкой системе сохраняется, непрерывность потока также существует.В этом уравнении A — площадь поперечного сечения потока, а v — скорость. Если площадь поперечного сечения в жидкостной системе изменится, скорость изменится обратно пропорционально, чтобы сохранить непрерывность.

3. Уравнение Бернулли: P + ½ρv 2 + ρgh = константа

  • Это уравнение позволяет анализировать движение жидкости по трубе и связывает скорость жидкости с ее давлением. Для горизонтальной трубы с изменяющимся диаметром области, в которых жидкость движется быстро, будут испытывать меньшее давление, чем области, в которых жидкость движется медленно.Уравнение Бернулли применяет принципы сохранения энергии к текущей жидкости. В этом уравнении P — гидростатическое давление, ρ — плотность жидкости, v — скорость, g — ускорение свободного падения (9,8 м/с 2 ), а h — высота жидкости в метрах.

4. Закон идеального газа: PV = nRT

  • Закон идеального газа описывает поведение идеального газа и объединяет идеи, найденные в различных других газовых законах. В этом уравнении P — давление газа, V — объем в литрах, n — количество газа в молях, R — универсальная газовая постоянная, а T — температура в градусах Кельвина.Значение R будет зависеть от единиц, которые вы используете в этом уравнении.

5. Закон Бойля: PV = константа, P 1 V 1 = P 2 V 2

  • Этот газовый закон гласит, что давление (P) газа находится в обратной зависимости от его объема. (V) при постоянной температуре. Закон Бойля позволяет рассчитать, как изменится объем газа при изменении оказываемого на него давления, и наоборот.

6. Закон Шарля: V/T = константа, V 1 /T 1 = V 2 /T 2

  • Этот газовый закон гласит, что объем (V) газа равен напрямую связана с его температурой (T) при постоянном давлении.Закон Шарля позволяет рассчитать, как изменится объем газа при изменении его температуры и наоборот.

7. Закон Авогадро: V/n = константа, V 1 /n 1 = V 2 /n 2

  • Этот газовый закон связывает количество молей газа внутри газа. Объем (V) газа напрямую связан с количеством молей (n) в нем. При постоянной температуре и давлении большее число молей будет занимать больший объем.Закон Авогадро позволяет рассчитать, как изменится объем газа при изменении количества молей, и наоборот.

8. Закон Дальтона для парциальных давлений: P Total = P 1 + P 2

  • отдельных давлений (P 1 , P 2 и т. д.), оказываемых каждым газом в смеси.

4C – Электрохимия и электрические цепи и их элементы

В этой категории контента делается акцент на природе электрических токов и напряжений, на том, как энергия может быть преобразована в электрические формы, которые можно использовать для выполнения химических превращений или работы.Кроме того, в эту категорию входит то, как электрические импульсы могут передаваться на большие расстояния в нервной системе.

1. Закон Кулона: F = k∙(q 1 q 2 /r 2 )

  • Этот закон определяет силу взаимодействия между двумя электрически заряженными частицами. Электрическая сила (F) отталкивания или притяжения между частицами пропорциональна произведению зарядов (q) и обратно пропорциональна квадрату расстояния между ними (r 2 ).В этом уравнении k — постоянная Кулона.

2. Постоянный ток: I = ΔQ/Δt

  • Это уравнение позволяет рассчитать электрический ток (I) в цепи, когда электрический заряд (ΔQ) течет в течение времени Δt.

3. Закон Ома: I = V/R

  • Закон Ома связывает ток (I), протекающий по цепи, с напряжением (V) и сопротивлением (R). Ток равен напряжению, деленному на сопротивление в омах.

4.Удельное сопротивление: ρ = R∙A/L

  • Это уравнение удельного сопротивления показывает, что удельное сопротивление (ρ) материала, такого как провод, равно сопротивлению (R) материала в омах, умноженному на его поперечное сечение. площади (A) и разделенной на его длину (L).

4D — Как свет и звук взаимодействуют с материей

Эта категория контента посвящена свойствам света и звука, тому, как взаимодействие света и звука с материей может использоваться организмом для восприятия окружающей среды и как эти взаимодействия также можно использовать для создания структурной информации или изображений.

1. Энергия фотона: E = hf

  • Энергия (E) фотона в электромагнитной волне напрямую связана с частотой волны (f). В этом уравнении h — постоянная Планка.

2. Закон Снеллиуса: n 1 sinθ 1 = n 2 sinθ 2

  • n 1 ) в другую среду с другим показателем преломления (n 2 ).Угол (sinθ 1 ) падения на поверхность и угол (sinθ 2 ) преломления измеряются относительно нормали к поверхности.

3. Уравнение линзы: 1/f = 1/p + 1/q

  • Искривление световых лучей через тонкую линзу описывается уравнением линзы. В этом уравнении f — фокусное расстояние линзы, p — расстояние объекта от линзы, а q — расстояние изображения от линзы. Вам нужно будет знать правила знаков для этого уравнения, или когда определенные значения будут положительными или отрицательными: для выпуклой линзы фокусное расстояние всегда будет положительным, для вогнутой линзы фокусное расстояние всегда будет отрицательным.

4E – Атомы, ядерный распад, электронная структура и атомно-химическое поведение конкретный атом можно использовать для предсказания его физических и химических свойств.

  • AAMC не ссылается на какие-либо конкретные физические уравнения, которые вам необходимо знать для этой последней категории контента в разделе «Химические и физические основы биологических систем» MCAT.

Если вы чувствуете себя ошеломленным количеством физических уравнений, которые вам нужно знать для MCAT, обязательно ознакомьтесь с нашими полезными советами ниже. Чтобы узнать о средних баллах и процентильных рангах для разделов химии и физики MCAT, загляните в наш блог Насколько сложно пройти MCAT?

Хотите узнать о лучшем графике обучения MCAT? Посмотрите наше видео:

Советы по использованию уравнений физики во время MCAT

Совет № 1. Помните, что вам не нужно быть гением в физике, чтобы хорошо сдать MCAT

Да, есть немало физические уравнения, которые вам нужно будет запомнить и досконально понять, как их использовать для MCAT, но это лишь малая часть физических уравнений, существующих во Вселенной.Они также не являются самыми сложными физическими уравнениями и обычно применяются к задачам, которые можно решить всего за несколько шагов. Вопросы в разделе химии и физики MCAT будут вращаться вокруг простых физических уравнений и основополагающих концепций. Ключ в том, чтобы понять, когда использовать эти уравнения и как использовать их быстро и уверенно. После запоминания каждого физического уравнения, которое вам необходимо знать, выполнение как можно большего количества практических задач MCAT по химии и физике поможет вам понять, как применять эти уравнения.Имейте в виду, что физические уравнения, которые вам понадобятся, просты: если вы обнаружите, что решаете сложную многошаговую задачу, и вы уже потратили несколько минут на расчеты, вам нужно пересмотреть свой подход.

Совет № 2: следите за единицами

Мы все сталкивались с этим: вы только что потратили пять минут на длительные расчеты и, взглянув на варианты ответов, увидели, что вашего решения нет среди возможных ответов. Вы начинаете паниковать и переживать, что только что потеряли пять драгоценных минут и до сих пор не знаете ответа.Часто быстрое преобразование единиц измерения может дать правильный ответ; или вы, возможно, просто использовали неправильные единицы измерения в своем уравнении. Понимание того, как конвертировать между единицами измерения, и обеспечение того, чтобы вы могли сделать это быстро без калькулятора, имеет важное значение для раздела химии и физики MCAT! Еще один совет: почувствуйте себя комфортно, перестраивая уравнения для решения конкретной переменной, чтобы избежать ошибок в день тестирования.

Совет №3: Примените свои знания физики

Концепции физики будут проверены в контексте живых систем.Поэтому типы вопросов, которые вы, возможно, встречали на вступительных экзаменах по физике в колледже, скорее всего, не появятся в MCAT. Не будет глубоких 30-минутных расчётов физики. Важно понимать, что вы будете применять фундаментальные понятия физики к человеческому телу, например, к пассажу о течении жидкости через аорту. При изучении физических концепций для MCAT сосредоточьтесь на применении этих физических концепций к человеческому телу. Если вы не знаете, как какая-либо физическая концепция применима к живым системам, вам стоит изучить это.

Чтобы получить дополнительные советы по MCAT, не забудьте также использовать наши вопросы по психологии и социологии MCAT, MCAT CARS, биологии MCAT и советы по биохимии, специально предназначенные для прохождения каждого раздела MCAT! Не забудьте ознакомиться с нашей надежной стратегией MCAT CARS!

Ознакомьтесь с кратким обзором:

Часто задаваемые вопросы

1. Какова продолжительность раздела химии и физики MCAT и каков его формат?

Секция химии и физики является первой из четырех секций MCAT.В этом разделе у вас будет 95 минут, чтобы ответить на 59 вопросов. Из этих 59 вопросов 44 основаны на отрывках. Вам будет представлено десять отрывков по темам химии и физики, и после каждого отрывка вам будет задано от четырех до семи вопросов, основанных на отрывке. Также будет 15 отдельных дискретных вопросов, разбросанных между отрывками. Заинтересованы в подробной разбивке того, как будет распределяться каждая минута в день тестирования? Посетите наш блог «Как долго длится MCAT?»

2. Как я могу использовать диагностический экзамен, чтобы определить, сколько физики мне нужно будет изучить для MCAT?

Прежде чем вы сможете начать подготовку к MCAT, вам необходимо понять свой базовый уровень. Для этого необходимо пройти полноценный диагностический тест MCAT. Цель состоит в том, чтобы точно понять, где вы находитесь, когда начинаете подготовку к MCAT. Для диагностики лучше всего использовать полноценный экзамен с веб-сайта AAMC. Убедитесь, что вы сдали практический экзамен за один присест в среде, которая имитирует условия тестового дня.Анализируя результаты диагностики, оцените свои сильные стороны и области, требующие улучшения. Как вы справились с вопросами, связанными с физикой? Вы ничего не поняли, когда дело дошло до определенных физических уравнений или областей содержания? Удалось ли вам связать свои знания по физике с вопросами о живых организмах и системах организма? Будьте честны с собой в отношении вашего уровня комфорта с физикой MCAT, когда вы просматриваете наш блог, который помогает вам определить общий вопрос «когда мне следует сдавать MCAT?» Руководство по расписанию учебы MCAT.

3. Какие методы запоминания физических уравнений мне понадобятся в день экзамена?

Во время подготовки к MCAT вы можете обнаружить, что традиционные методы запоминания уравнений, такие как создание карточек, вам не подходят. Что еще вы можете попробовать? Вот некоторые дополнительные приемы, которые можно использовать при подготовке к MCAT:

  • Запишите уравнение несколько раз на листе бумаги, пока вы не сможете произнести его вслух, не обращаясь к своим учебным материалам.
  • Попробуйте расширить уравнение до предложения, объясняющего, о чем оно говорит.
  • Решите несколько практических задач, требующих использования уравнения.
  • Попробуйте сгруппировать несколько уравнений по темам, чтобы увидеть сходство между уравнениями, с которыми вы сталкиваетесь, и теми, которые вы уже освоили.
  • Спросите друга, разработали ли они какие-нибудь броские мнемонические приемы для запоминания физических уравнений, которые вам понадобятся для MCAT.

Помните, правильное понимание уравнения является ключевым моментом в его запоминании.Для любых уравнений, с которыми вы боретесь, углубитесь в каждую часть уравнения и постарайтесь понять, как каждая часть работает вместе. Вы также можете попробовать вернуться к своим заметкам и просмотреть любые уравнения, относящиеся к основополагающим понятиям, которые вы узнали ранее. Пробелы в знаниях по темам, которые вы уже рассмотрели, могут мешать вам изучать новые уравнения. Если вы действительно боретесь, вы можете обратиться к репетитору MCAT.

4. Могу ли я использовать калькулятор при решении физических уравнений на MCAT?

Вы не сможете использовать калькулятор ни в одном из разделов MCAT, а это означает, что важно, чтобы вы не заполняли примеры вопросов и не практиковались на экзаменах MCAT с помощью калькулятора.Важно настроить себя на успех, завершив подготовку к MCAT в условиях, которые будут имитировать условия дня экзамена. Используйте месяцы, предшествующие экзамену MCAT, чтобы повысить эффективность выполнения расчетов в уме и математических расчетов вручную.

5. Нужно ли мне знать, как рисовать схемы свободного тела для MCAT?

MCAT представляет собой тест с несколькими вариантами ответов и не содержит вопросов со свободным ответом, на которых проверяются ваши знания о том, как рисовать диаграммы.При этом базовые знания о том, как рисовать диаграммы свободного тела для расчета сил, безусловно, пригодятся при решении вопросов с несколькими вариантами ответов, связанных с физикой, поэтому не пренебрегайте отработкой этого навыка.

6. Достаточно ли AP Physics для решения вопросов MCAT по физике?

Ответ на этот вопрос, конечно же, будет зависеть от того, насколько хорошо вы справились с курсом физики AP. AP Physics должен дать вам те же вводные знания по физике, которые вы получили бы на вводном курсе физики в университете.Используйте свой диагностический экзамен, чтобы действительно оценить свой уровень комфорта с помощью уравнений физики и концепций физики на MCAT. Если у вас есть хороший базовый балл по разделу химии и физики MCAT, вы можете опираться на свои знания по физике AP, убедившись, что вы по-прежнему знаете необходимые уравнения, и закрепляя ключевые понятия по мере изучения. Скорее всего, вам все равно придется исследовать, как фундаментальные концепции физики соотносятся с живыми системами, так как это, возможно, не было подчеркнуто в вашем курсе физики AP.

7. Почему только эти уравнения указаны как основные? Разве мне не нужно знать больше?

Да, вам могут понадобиться другие, но вы можете легче вывести или вывести их на основе этих основных. Если вы не знаете этих основных, вы не сможете вывести или вывести остальные.

8. Когда мне следует сдавать MCAT?

Обязательно сдайте экзамен, когда будете готовы. Обычно это означает, что вы постоянно набираете 90-й процентиль на практических экзаменах.

Заключение

Почему физика включена в MCAT? Физика — лишь один из многих строительных блоков, которые понадобятся вам как студенту-медику, чтобы узнать о физиологических функциях дыхательной, сердечно-сосудистой и неврологической систем в норме и при болезни. Таким образом, раздел химии и физики MCAT — это ваша возможность продемонстрировать свое понимание того, как фундаментальные концепции физики будут применяться к вашей будущей карьере в медицине. Ключевым элементом вашего успеха на MCAT будет начать изучать и по-настоящему понимать каждое из физических уравнений, описанных в этом блоге.

За ваш успех,

Ваши друзья в BeMo

BeMo Academic Consulting

6.3 Вращательное движение — физика

Раздел Цели обучения

К концу этого раздела вы сможете делать следующее:

  • Описать вращательные кинематические переменные и уравнения и связать их с их линейными аналогами
  • Опишите крутящий момент и плечо рычага
  • Решение задач, связанных с крутящим моментом и кинематикой вращения

Поддержка учителей

Поддержка учителей

Цели обучения в этом разделе помогут вашим ученикам освоить следующие стандарты:

  • (4) Научные концепции.Учащийся знает и применяет законы, управляющие движением, в различных ситуациях. Ожидается, что студент:
    • (C) анализировать и описывать ускоренное движение в двух измерениях, используя уравнения, включая примеры снарядов и окружностей.
    • (D) рассчитать действие сил на объекты, включая закон инерции, взаимосвязь между силой и ускорением и характер пар сил между объектами.

Кроме того, в Руководстве по физике для старших классов рассматривается содержание этого раздела лабораторной работы под названием «Круговое и вращательное движение», а также следующие стандарты:

  • (4) Научные концепции.Учащийся знает и применяет законы, управляющие движением, в различных ситуациях. Ожидается, что студент:
    • (D) рассчитать действие сил на объекты, включая закон инерции, связь между силой и ускорением и природу пар сил между объектами.

Основные термины раздела

угловое ускорение кинематика вращательного движения рычаг
тангенциальное ускорение крутящий момент

Кинематика вращения

Служба поддержки учителей

Служба поддержки учителей

[BL][OL] Повторить уравнения линейной кинематики.

Предупреждение о неправильном представлении

Учащиеся могут запутаться между замедлением и увеличением ускорения в отрицательном направлении.

В разделе, посвященном равномерному круговому движению, мы обсуждали движение по окружности с постоянной скоростью и, следовательно, с постоянной угловой скоростью. Однако бывают случаи, когда угловая скорость непостоянна — вращательное движение может ускоряться, замедляться или изменять направление. Угловая скорость не является постоянной, когда вращающийся фигурист тянет руки, когда ребенок толкает карусель, чтобы заставить ее вращаться, или когда компакт-диск останавливается при выключении.Во всех этих случаях угловое ускорение возникает из-за изменения угловой скорости ωω. Чем быстрее происходит изменение, тем больше угловое ускорение. Угловое ускорение αα – скорость изменения угловой скорости. В форме уравнения угловое ускорение равно

где ΔωΔω — изменение угловой скорости, а ΔtΔt — изменение во времени. Единицы углового ускорения: (рад/с)/с или рад/с 2 . Если ωω увеличивается, то αα положительно. Если ωω уменьшается, то αα отрицательно.Имейте в виду, что по соглашению против часовой стрелки — это положительное направление, а по часовой стрелке — отрицательное направление. Например, фигуристка на рис. 6.9 вращается против часовой стрелки, если смотреть сверху, поэтому ее угловая скорость положительна. Ускорение будет отрицательным, например, когда объект, вращающийся против часовой стрелки, замедляется. Было бы положительно, если бы объект, вращающийся против часовой стрелки, ускорялся.

Рис. 6.9 Фигуристка вращается против часовой стрелки, поэтому ее угловая скорость обычно считается положительной.(Luu, Wikimedia Commons)

Соотношение между величинами тангенциального ускорения, a , и углового ускорения,

α,isa=rαorα=ar.α,isa=rαorα=ar.

6.10

Эти уравнения означают, что величины тангенциального ускорения и углового ускорения прямо пропорциональны друг другу. Чем больше угловое ускорение, тем больше изменение тангенциального ускорения, и наоборот. Например, рассмотрим всадников в своих капсулах на колесе обозрения в состоянии покоя.Колесо обозрения с большим угловым ускорением даст гонщикам большее тангенциальное ускорение, потому что, поскольку колесо обозрения увеличивает скорость вращения, оно также увеличивает свою тангенциальную скорость. Обратите внимание, что радиус вращающегося объекта также имеет значение. Например, для данного углового ускорения αα меньшее колесо обозрения приводит к меньшему тангенциальному ускорению для гонщиков.

Советы для достижения успеха

Тангенциальное ускорение иногда обозначается как a t .Это линейное ускорение в направлении, касательном к окружности в интересующей точке при круговом или вращательном движении. Помните, что тангенциальное ускорение параллельно тангенциальной скорости (либо в том же направлении, либо в противоположном направлении). Центростремительное ускорение всегда перпендикулярно тангенциальной скорости.

До сих пор мы определили три вращательные переменные: θθ, ωω и αα. Это угловые версии линейных переменных x , v и a .Таблица 6.2 показывает, как они связаны.

Поворотный Линейный Отношения
θθ х θ=xrθ=xr
ωω v ω=врω=вр
αα и α=арα=ар

Таблица 6. 2 Вращательные и линейные переменные

Теперь мы можем начать видеть, как вращательные величины, такие как θθ, ωω и αα, связаны друг с другом. Например, если колесо мотоцикла, находящееся в состоянии покоя, имеет большое угловое ускорение в течение достаточно долгого времени, оно в конечном итоге начинает быстро вращаться и делает много оборотов. Выражая это в терминах переменных, если угловое ускорение колеса αα велико в течение длительного периода времени t , то конечная угловая скорость ωω и угол поворота θθ велики.В случае линейного движения, если объект находится в состоянии покоя и испытывает большое линейное ускорение, то он имеет большую конечную скорость и пройдёт большое расстояние.

Кинематика вращательного движения описывает соотношения между углом поворота, угловой скоростью, угловым ускорением и временем. Это только описывает движение — оно не включает никаких сил или масс, которые могут повлиять на вращение (это часть динамики). Вспомните уравнение кинематики для линейного движения: v=v0+atv=v0+at (константа a ).

Как и в линейной кинематике, мы предполагаем, что a является постоянным, что означает, что угловое ускорение αα также является постоянным, поскольку a=rαa=rα. Уравнение кинематической зависимости между ωω, αα и t равно

ω=ω0+αt(константаα),ω=ω0+αt(константаα),

, где ω0ω0 — начальная угловая скорость. Обратите внимание, что уравнение идентично линейной версии, за исключением угловых аналогов линейных переменных. Фактически все уравнения линейной кинематики имеют вращательные аналоги, которые приведены в таблице 6.3. Эти уравнения можно использовать для решения вращательной или линейной задачи кинематики, в которой a и αα являются постоянными.

Поворотный Линейный
θ=ω¯tθ=ω¯t х=v¯tx=v¯t
ω=ω0+αtω=ω0+αt v=v0+αtv=v0+αt константа αα, a
θ=ω0t+12αt2θ=ω0t+12αt2 х=v0t+12αt2x=v0t+12αt2 константа αα, a
ω2=ω02+2αθω2=ω02+2αθ v2=v02+2αxv2=v02+2αx константа αα, a

Таблица 6. 3 Уравнения вращательной кинематики

В этих уравнениях ω0ω0 и v0v0 — начальные значения, t0t0 равно нулю, а средняя угловая скорость ω¯ω¯ и средняя скорость v¯v¯ равны

ω¯=ω0+ω2иv¯=v0+v2.ω¯=ω0+ω2иv¯=v0+v2.

6.11

Fun In Physics

Погоня за штормом

Рис. 6.10. Торнадо опускаются из облаков в виде воронок, которые сильно вращаются. (Дафна Зарас, Национальное управление океанических и атмосферных исследований США)

Охотники за штормами, как правило, попадают в одну из трех групп: любители, гоняющиеся за торнадо в качестве хобби, атмосферные ученые, собирающие данные для исследований, наблюдатели за погодой для средств массовой информации или ученые, развлекающиеся под видом работы.Погоня за штормом — опасное времяпрепровождение, потому что торнадо может быстро изменить курс без малейшего предупреждения. Поскольку за разрушениями, оставленными торнадо, следуют охотники за штормами, замена спущенных шин из-за обломков, оставленных на шоссе, является обычным явлением. Наиболее активная часть мира для торнадо, называемая переулком торнадо , находится в центральной части Соединенных Штатов, между Скалистыми горами и Аппалачами.

Торнадо — прекрасный пример вращательного движения в действии в природе.Они возникают во время сильных гроз, называемых суперячейками, которые имеют столб воздуха, вращающийся вокруг горизонтальной оси, обычно около четырех миль в поперечнике. Разница в скорости ветра между сильными холодными ветрами выше в атмосфере в струйном течении и более слабыми ветрами, движущимися на север от Мексиканского залива, заставляет столб вращающегося воздуха смещаться так, что он вращается вокруг вертикальной оси, создавая торнадо.

Торнадо создают скорость ветра до 500 км/ч (примерно 300 миль/ч), особенно внизу, где воронка самая узкая, поскольку скорость вращения увеличивается по мере уменьшения радиуса.Они сдувают дома, как если бы они были сделаны из бумаги, и, как известно, протыкают стволы деревьев кусочками соломы.

Проверка захвата

Какой физический термин обозначает глаз бури? Почему ветры в эпицентре торнадо слабее, чем на его внешнем краю?

  1. Глаз бури — центр вращения. Ветры слабее в глазу торнадо, потому что тангенциальная скорость прямо пропорциональна радиусу кривизны.
  2. Глаз бури — центр вращения.Ветры слабее в глазу торнадо, потому что тангенциальная скорость обратно пропорциональна радиусу кривизны.
  3. Глаз бури — центр вращения. Ветры слабее в глазу торнадо, потому что тангенциальная скорость прямо пропорциональна квадрату радиуса кривизны.
  4. Глаз бури — центр вращения. Ветры слабее в глазу торнадо, потому что тангенциальная скорость обратно пропорциональна квадрату радиуса кривизны.

Крутящий момент

Если вы когда-нибудь крутили велосипедное колесо или толкали карусель, вы знаете, что для изменения угловой скорости необходима сила. Чем дальше сила приложена от точки поворота (или точки опоры), тем больше угловое ускорение. Например, дверь открывается медленно, если вы нажимаете слишком близко к петле, но открывается легко, если вы нажимаете далеко от петли. Кроме того, мы знаем, что чем массивнее дверь, тем медленнее она открывается; это потому, что угловое ускорение обратно пропорционально массе.Эти отношения очень похожи на отношения между силой, массой и ускорением из второго закона Ньютона. Поскольку мы уже рассмотрели угловые версии расстояния, скорости и времени, вы можете задаться вопросом, что такое угловая версия силы и как она соотносится с линейной силой.

Угловой версией силы является крутящий момент ττ, который представляет собой поворачивающую эффективность силы. См. Рисунок 6.11. Уравнение для величины крутящего момента:

, где r — величина плеча рычага, F — величина линейной силы, а θθ — угол между плечом рычага и силой. Плечо рычага — это вектор от точки вращения (точка вращения или точка опоры) к месту приложения силы. Поскольку величина плеча рычага представляет собой расстояние, его единицы измерения — метры, а крутящий момент — Н⋅м. Крутящий момент является векторной величиной и имеет то же направление, что и создаваемое им угловое ускорение.

Рис. 6.11 Человек толкает карусель за ее край и перпендикулярно плечу рычага для достижения максимального крутящего момента.

Применение большего крутящего момента приведет к большему угловому ускорению.Например, чем сильнее человек толкает карусель на рис. 6.11, тем быстрее она ускоряется. Кроме того, чем массивнее карусель, тем медленнее она разгоняется при том же крутящем моменте. Если человек хочет максимизировать воздействие своей силы на карусель, он должен толкать ее как можно дальше от центра, чтобы получить наибольшее плечо рычага и, следовательно, наибольший крутящий момент и угловое ускорение. Крутящий момент также максимизируется, когда сила приложена перпендикулярно плечу рычага.

Поддержка учителя

Поддержка учителя

[BL][OL][AL] Продемонстрируйте физические взаимосвязи между крутящим моментом, силой, углом приложения силы и длиной плеча рычага, используя рычаги разной длины. Помогите учащимся установить связь между физическими наблюдениями и математическими соотношениями. Например, крутящий момент максимален, когда сила приложена точно перпендикулярно плечу рычага, потому что sinθ=1sinθ=1 для θ=90θ=90 градусов.

Решение задач кинематики вращения и крутящего момента

Точно так же, как линейные силы могут уравновешиваться, создавая нулевую результирующую силу и линейное ускорение, то же самое верно и для вращательного движения.Когда два крутящих момента одинаковой величины действуют в противоположных направлениях, нет ни чистого крутящего момента, ни углового ускорения, как вы можете видеть в следующем видео. Если нулевой чистый крутящий момент действует на систему, вращающуюся с постоянной угловой скоростью, система будет продолжать вращаться с той же угловой скоростью.

Watch Physics

Введение в крутящий момент

В этом видеоролике крутящий момент определяется с точки зрения плеча момента (которое совпадает с плечом рычага). Он также охватывает проблему с силами, действующими в противоположных направлениях вокруг точки поворота.(На этом этапе вы можете игнорировать упоминания Сала о работе и механических преимуществах.)

Проверка захвата

Если бы чистый крутящий момент, действующий на линейку из примера, был бы положительным, а не нулевым, что бы это сказало об угловом ускорении? Что произойдет с правителем со временем?

  1. Линейка находится в состоянии вращательного равновесия, поэтому она не будет вращаться вокруг своего центра масс. Таким образом, угловое ускорение будет равно нулю.
  2. Линейка не находится в состоянии вращательного равновесия, поэтому она не будет вращаться вокруг своего центра масс. Таким образом, угловое ускорение будет равно нулю.
  3. Линейка не находится в состоянии вращательного равновесия, поэтому она будет вращаться вокруг своего центра масс. Таким образом, угловое ускорение будет ненулевым.
  4. Линейка находится в состоянии вращательного равновесия, поэтому она будет вращаться вокруг своего центра масс. Таким образом, угловое ускорение будет ненулевым.

Теперь давайте рассмотрим примеры применения вращательной кинематики к рыболовной катушке и концепции крутящего момента к карусели.

Рабочий пример

Расчет времени остановки вращения рыболовной катушки

Глубоководный рыбак использует удочку с катушкой радиусом 4,50 см. Большая рыба берет наживку и уплывает от лодки, вытягивая леску из своей рыболовной катушки. По мере разматывания лески с катушки катушка вращается с угловой скоростью 220 рад/с. Рыбак тормозит спиннинговую катушку, создавая угловое ускорение −300 рад/с 2 .Сколько времени требуется барабану, чтобы остановиться?

Стратегия

Нас просят найти время t для остановки барабана. Величина начальной угловой скорости ω0=220ω0=220 рад/с, а величина конечной угловой скорости ω=0ω=0 . Величина углового ускорения со знаком равна α=−300α=−300 рад/с 2 , где знак минус указывает на то, что оно действует в направлении, противоположном угловой скорости.Глядя на кинематические уравнения вращения, мы видим, что все величины, кроме t , известны в уравнении ω=ω0+αtω=ω0+αt, что делает его самым простым уравнением для решения этой задачи.

Решение

Используемое уравнение: ω=ω0+αtω=ω0+αt .

Алгебраически решаем уравнение для t , а затем подставляем известные значения.

t=ω-ω0α=0-220рад/с-300рад/с2=0,733st=ω-ω0α=0-220рад/с-300рад/с2=0.733s

6.12

Обсуждение

Время остановки барабана довольно мало, потому что ускорение довольно велико. Леска иногда рвется из-за прилагаемой силы, и рыбаки часто позволяют рыбе немного поплавать, прежде чем затормозить катушку. Усталая рыба будет медленнее, ей потребуется меньшее ускорение и, следовательно, меньшая сила.

Рабочий пример

Расчет крутящего момента карусели

Рассмотрим человека, толкающего игровую карусель на рис. 6.11. Он прикладывает силу 250 Н на краю карусели и перпендикулярно радиусу, который равен 1,50 м. Какой крутящий момент он выдает? Предположим, что трение, действующее на карусель, пренебрежимо мало.

Стратегия

Чтобы найти крутящий момент, обратите внимание, что приложенная сила перпендикулярна радиусу и что трением можно пренебречь.

Решение

τ=rFsinθ=(1,50м)(250Н)sin(π2).=375Н⋅мτ=rFsinθ=(1,50м)(250Н)sin(π2).=375Н⋅м

6.13

Обсуждение

Мужчина максимизирует крутящий момент, прикладывая силу перпендикулярно плечу рычага, так что θ=π2θ=π2 и sinθ=1sinθ=1 . Мужчина также максимизирует свой крутящий момент, нажимая на внешний край карусели, так что он получает максимально возможное плечо рычага.

Практические задачи

15.

Какой крутящий момент создаст человек, если он приложит силу 12\,\text{N} на расстоянии 1,0\,\text{м} от точки вращения, перпендикулярно плечу рычага?

  1. \frac{1}{144}\,\text{Н-м}
  2. \frac{1}{12}\,\text{Н-м}
  3. 12\,\text{Н-м}
  4. 144\,\text{Н-м}
16.

Угловая скорость объекта изменяется с 3 рад/с по часовой стрелке до 8 рад/с по часовой стрелке за 5 с. Каково его угловое ускорение?

  1. 0,6 рад/с 2
  2. 1,6 рад/с 2
  3. 1 рад/с 2
  4. 5 рад/с 2

Проверьте свое понимание

17.

Что такое угловое ускорение?

  1. Угловое ускорение — это скорость изменения углового смещения.
  2. Угловое ускорение — скорость изменения угловой скорости.
  3. Угловое ускорение — это скорость изменения линейного перемещения.
  4. Угловое ускорение — скорость изменения линейной скорости.
18.

Какое уравнение для углового ускорения, α ? Предположим, что θ — это угол, ω — это угловая скорость, а t — это время.

  1. α=ΔωΔtα=ΔωΔt
  2. α=ΔωΔtα=ΔωΔt
  3. α=ΔθΔtα=ΔθΔt
  4. α=ΔθΔtα=ΔθΔt
19.

Что из следующего лучше всего описывает крутящий момент?

  1. Это вращательный эквивалент силы.
  2. Это сила, влияющая на линейное движение.
  3. Это вращательный эквивалент ускорения.
  4. Это ускорение влияет на линейное движение.
20.

Какое уравнение для крутящего момента?

  1. \тау = {F\,cos\тета}\,{r}
  2. \тау = \фрак{F\sin\theta}{r}
  3. \тау = rF\!\cos\тета
  4. \тау = rF\!\sin\тета

Служба поддержки учителей

Служба поддержки учителей

Используйте вопросы «Проверьте свое понимание», чтобы оценить, справляются ли учащиеся с целями обучения в этом разделе. Если учащиеся борются с определенной целью, эти вопросы помогут определить, какая цель вызывает проблему, и направить учащихся к соответствующему содержанию.

CH 12,13,14,15 16-Ready -2017 Springula Performula Pixula Pientula — Engr 2302 —

Формулас и

Глава12

Средняя





  (всегдаspositivenumbers)  



 



(МолчанБозвана, солизирующаясяOrzero) 

Processilemotion

 (x -Дерекция)  (Y-направление) 

 

  







2󰇛󰇜

CH12.kinematicRelationsships: RECTILINEARMOTION

 (ускорениеКонационно)   (ускорениеConstant) 

































󰇛

󰇜   









Ch12. kineMaticRelationsships: generalmotionincartesiancoordinates

(скорость)  (ускорение) 

󰇗 󰇘

󰇗󰇘

󰇗 󰇘

=> FindmagnitudesfrompythagoreanTheRem.forexample: 







Ch12.КинематическиеВзаимосвязи:ОбщиеДвижениевНормально-тангенциальное(n-t)Координаты

(Скорость)

󰇗



󰇗









=> :

󰇡





󰇢

 / 

 





Радиусизкривизны,y(x)ist онтраектория

Ch12.kinematicrelationsssss: generalmotioninpolarcoordinates (2dcase) 

(скорость)  (ускорение) 

󰇗 

󰇘󰇗

󰇗

󰇘 󰇗󰇗

󰇛󰇜

󰇛󰇜

󰇛󰇜



 󰇛󰇜

󰇛󰇜

󰇛󰇜









Физика 581: Обучение динамике с Excel90

Физика 581 Часть 1: Обучение динамике с помощью Excel97

Нажмите здесь по физике 581 часть 2

Майкл Фаулер, Уфа, лето 1998 г.

Введение

В отличие от курса Галилея и Эйнштейна материал представленные здесь предназначены для непосредственного использования учителем или учеником в более традиционный курс физики средней школы или начальный университетский курс на механика. Мы все еще используем часть материала Galileo и Einstein , но мы усиливаем многие темы и обогащаем микс некоторыми электронными таблицами.

По-настоящему понять ньютоновскую динамику требует некоторого понимания его величайшее изобретение исчисления.Вот где электронные таблицы бесценны. От построить электронную таблицу, чтобы найти скорость как функцию времени, принимая различия положения в последовательные промежутки времени, и продолжая делать то же самое для ускорения, студент (по моему опыту) выстраивает более четкую представление о значении производной, чем то, которое обычно получается из формальных математических манипуляции. Создание этих таблиц занимает некоторое время, но я даю очень подробная инструкция. Это, по общему признанию, приводит к опасности того, что студент будет слепо строить электронную таблицу, не понимая ее по-настоящему, но понимание придет после достаточного количества упражнений: пусть учащиеся зарисуют разные вещей, варьировать зависимость силы сопротивления от скорости и, особенно, варьировать параметры (включая количество строк) пока таблица не выдаст чушь, тогда попробуй понять почему!

Готовые таблицы некоторых упражнений я выложил в сеть, чтобы их можно было скачать и изучить.

Книга:

В этом курсе мы просто даем пример некоторых способов использования электронных таблиц в физика. Если вы хотите увидеть более широкий спектр более увлекательного материала, я настоятельно рекомендуем приобрести «Физику электронных таблиц» Миснера и Куни (около 28 долларов, я думаю). Я купил его в 1992 году и использовал некоторые из идеи, такие как интеграция чехарды и простой маятник в качестве теста Эйлер против чехарды. Одной из неприятных особенностей книги является то, что она никогда не был обновлен, поэтому он настроен для Lotus 1-2-3 на базе DOS, но идеи отличный. Был доступен диск с шаблонами, но я его не получил, поэтому не знаю. знаю, будет ли это все еще работать с Excel.

Начало Динамика: одномерное движение

Первые шаги в динамике — это реальное понимание концепций скорости и ускорение ,

и все учителя физики знают, что это невероятно сложно! Цель Первые три лекции должны понять скорость и ускорение в одномерное движение, с либеральным количественным использованием видео камера и электронная таблица.Мы делаем это до введения законов Ньютона, но в Анализируя сопротивление воздуха, мы предполагаем, что при постоянной конечной скорости воздух Сила сопротивления сопротивления уравновешивает вес, чтобы найти, как сила сопротивления зависит от скорость (для пакетов кофейных фильтров).

1: Как быстро Падающий мяч падает?

В первой лекции мы рассматриваем объекты, падающие вертикально под действием силы тяжести. Это движение изучалось с древних времен, и Аристотель пытался проанализировать движение количественно. Он был блестящим человеком, но он получил это неправильный. Вопрос в том, почему? Галилей справился намного лучше, но это было почти два тысячу лет спустя. Стоит задуматься над тем, что сделал Галилей с Аристотелем. не удалось сделать. Безусловно, стоит провести эксперимент с Галилеем, как описано в эта лекция.

2: Анализ видео падающего мяча с помощью Эксель.

Мы покажем, как измерять движение падающего мяча гораздо более непосредственно, используя видеокамеру и воспроизводя кадр за кадром, чтобы отслеживать мяч.Мы тогда введите наши результаты в электронную таблицу Excel, чтобы найти ускорение, вызванное сила тяжести.

3: Реальные эффекты: Сопротивление воздуха.

Видеотехника очень хорошо подходит для измерения сопротивления воздуха. последствия. Мы бросаем небольшие стопки кофейных фильтров и обнаруживаем, что они почти немедленно достичь конечной скорости, и смогли выяснить, что воздух сопротивление пропорционально квадрату скорости. В том числе эти Эффекты реального мира также делают точку зрения Аристотеля более понятной.

Подъем к Два измерения: снаряды, планеты и законы Ньютона

Идею скорости не так уж сложно понять в двух словах. размеры представлены стрелкой, вектор но ускорение равно очень трудно! И, если вы не понимаете досконально ускорение в два размеры, вы не понимаете, что такое Ньютон, вы не понимаете динамики. Итак, это очень важный .

4: Первые идеи о движении снаряда

Развитие идей Галилея о снарядах и драматические связь этих земных идей с движением Луны и Земли сам Исаак Ньютон, рассматривается в моем курсе по Галилею и Эйнштейну. 5, 6, 8, 9 ниже относятся к этому курсу, ссылки 7 и 10 дают дополнительный материал.

5: Векторы

6: Ньютон: от Снаряды до Луны

7: Силы, Равновесие, системы отсчета и законы Ньютона

8: Больше по законам Ньютона

9: Импульс, Работа и энергия

10:Подробнее об Потенциальная энергия

Использование Ньютона Законы для решения реальных проблем

Движения всех частиц от пылинок до планет на орбите дано с предельной точностью, применяя законы Ньютона. Если мы знаем силы испытывает частица, мы можем сразу найти ее ускорение, т. е. как его скорость изменяется во времени, и из этого мы можем построить карту его пути через пространство. Итак в принципе мы умеем решать уравнения движение. Но есть запись того, что мы только что сказали математически дает дифференциальное уравнение второго порядка для положения частиц в виде функция времени с (обычно) силой, зависящей от положения, и уравнением обычно это будет тот, который мы не знаем, как решить математически! Добро новость заключается в том, что эти уравнения могут быть решены численно (хотя это быстро становится более громоздким, если мы увеличим количество движущихся частицы).И вот где приходит электронная таблица: по крайней мере, для одной частицы проблемы, почти во всех ситуациях обычная электронная таблица имеет достаточную мощность, чтобы сюжетные траектории. Как мы увидим позже, метод становится ненадежным для очень быстро меняющиеся потенциалы, но это тоже стоит исследовать, потому что он дает представление о том, насколько можно доверять численным методам, и как их можно улучшить.

11. Исчисление Лечение падающего мяча сопротивлением воздуха

В этой первой «реальной» лекции мы не используем электронные таблицы, но решить две проблемы сопротивления воздуха (сопротивление, пропорциональное скорости и скорости в квадрате) аналитически.Эти решения предоставляют полезные тесты для проверки наших численные методы на точность и надежность. Мы можем найти, например, как точность нашей электронной таблицы зависит от количества используемых строк.

12. Электронная таблица Лечение падающего мяча сопротивлением воздуха

В этой лекции даны очень подробные инструкции по созданию таблица, решающая задачу о падении мяча с сопротивлением воздуха. То полная таблица доступна для скачивания ниже, но это ценная упражнение, чтобы построить его самостоятельно! Вы поймете это намного лучше и сможете адаптировать его для других задач.

Скачать Электронная таблица для падающего мяча с линейным сопротивлением воздуха

Скачать Электронная таблица для падающего мяча с сопротивлением воздуха, пропорциональным скорости

в квадрате

13. Электронная таблица Обработка снарядов с сопротивлением воздуху

Это гораздо более интересная задача, имеющая практическое значение. Приложения. Какова траектория снаряда при сопротивлении воздуха не заброшен? Какой угол проекции дает максимальную дальность, например? Этот таблица дает ответы!

Скачать Таблица для траектории снаряда с сопротивлением воздуха

14.Электронная таблица Лечение простым маятником

Это не просто гармонический осциллятор, а маятник с струна заменена стержнем, вынужденным вращаться в вертикальной плоскости, так что он может качаться «сверху». Чтобы решить это математически, требуется эллиптическая функции, но эта простая электронная таблица дает очень точный отчет о движение. И вы можете добавить сопротивление воздуха в качестве упражнения.

15. Из Простой маятник на планетарные орбиты

Начав с электронной таблицы Simple Pendulum, мы адаптируем ее к двумерный простой гармонический осциллятор, затем с небольшими изменениями в электронная таблица для орбит при потенциале обратных квадратов. Оба двумерных простой гармонический осциллятор и потенциал обратных квадратов имеют эллиптическую орбит, но мы можем видеть важные отличия от этих электронных таблиц.

Электронная таблица для планетарных орбит

16. Больше общих законов силы

Легко адаптировать таблицу планет, чтобы посмотреть на законы силы, другие чем обратный квадрат. Мы обнаруживаем, что орбиты становятся намного сложнее. Идеальная круговая орбита всегда возможна, но она нестабильна, за исключением сила обратных квадратов! Таким образом, существование Солнечной системы означает, что гравитационное притяжение обратно пропорционально квадрату.Вы можете изучить, как закон силы влияет на орбиты, загрузив таблицу ниже, но вы обнаружите, что из-за притяжения, намного более сильного, чем обратный квадрат, планета быстро падает к солнцу, чем движется так быстро, что численный анализ становится ненадежный.

Электронная таблица для более общих законов силы

Домашние задания и т.

Добавить комментарий

Ваш адрес email не будет опубликован. Обязательные поля помечены *